Решение матрицы методом крамера онлайн: Онлайн калькулятор. Решение систем линейных уравнений. Метод Крамера

Содержание

Решение системы линейных уравнений (метод Крамера)

Этот онлайн калькулятор позволит вам очень просто решить систему линейных уравнений (СЛУ) методом Крамера.

Для того чтобы решить систему линейных уравнений методом Крамера, выберите количество неизвестных величин: 2345

Заполните систему линейных уравнений

Для изменения в уравнении знаков с “+” на “-” вводите отрицательные числа. Если в вашем уравнение отсутствует какой-то коэффициент, то на его месте в калькуляторе введите ноль. Вводить можно числа или дроби. Например: 1.5 или 1/7 или -1/4 и т.д.

Решить систему

Воспользуйтесь также:
Решение системы линейных уравнений (метод подстановки)
Решение системы линейных уравнений (метод Гаусса)
Решение системы линейных уравнений (матричный метод)

Решение системы линейных уравнений методом Крамера

Метод Крамера

Метод Крамера – это метод решения квадратных систем линейных алгебраических уравнений с ненулевым определителем основной матрицы (то есть в случае, когда система уравнений имеет единственное решение). Основным математическим действием при решении системы уравнения методом Крамера является вычисление определителей матриц размерностью n (где n – количество уравнений в системе).

На нашем сайте вы можете решать системы уравнений методом Крамера в режиме онлайн. При этом решение вы получаете мгновенно, и оно является полным и подробным. При решении системы уравнений нужно находить определители нескольких разных матриц. Для сокращения решения эта операция упрощена (выдаётся лишь результат). Но вы можете при необходимости получить полное решение нахождения детерминанта матрицы. Соответствующий калькулятор имеется на нашем ресурсе.

Метод Крамера решения систем линейных уравнений

Метод Крамера основан на использовании определителей в решении систем линейных уравнений. Это значительно ускоряет процесс решения.

Метод Крамера может быть использован в решении системы стольких линейных уравнений, сколько в каждом уравнении неизвестных. Если определитель системы не равен нулю, то метод Крамера может быть использован в решении, если же равен нулю, то не может. Кроме того, метод Крамера может быть использован в решении систем линейных уравнений, имеющих единственное решение.

Определение. Определитель, составленный из коэффициентов при неизвестных, называется определителем системы и обозначается (дельта).

Определители

получаются путём замены коэффициентов при соответствующих неизвестных свободными членами:

;

.

Формулы Крамера для нахождения неизвестных:

.

Найти значения  и возможно только при условии, если

.

Этот вывод следует из следующей теоремы.

Теорема Крамера . Если определитель системы отличен от нуля, то система линейных уравнений имеет одно единственное решение, причём неизвестное равно отношению определителей. В знаменателе – определитель системы, а в числителе – определитель, полученный из определителя системы путём замены коэффициентов при этом неизвестном свободными членами. Эта теорема имеет место для системы линейных уравнений любого порядка.


Пример 1. Решить систему линейных уравнений:

.                         (2)

Согласно теореме Крамера имеем:

Итак, решение системы (2):

Для проверки решений систем уравнений 3 Х 3 и 4 Х 4 можно воспользоваться онлайн-калькулятором, решающим методом Крамера.


Как явствует из теоремы Крамера, при решении системы линейных уравнений могут встретиться три случая:

Первый случай: система линейных уравнений имеет единственное решение

(система совместна и определённа)

Условия:

*

Второй случай: система линейных уравнений имеет бесчисленное множество решений

(система совместна и неопределённа)

Условия:

* ,

** ,

т. е. коэффициенты при неизвестных и свободные члены пропорциональны.

Третий случай: система линейных уравнений решений не имеет

(система несовместна)

Условия:

*

** .

Итак, система m линейных уравнений с n переменными называется несовместной, если у неё нет ни одного решения, и совместной, если она имеет хотя бы одно решение. Совместная система уравнений, имеющая только одно решение, называется определённой, а более одного – неопределённой.

Пусть дана система

.

На основании теоремы Крамера


………….
,

где

определитель системы. Остальные определители получим, заменяя столбец с коэффициентами соответствующей переменной (неизвестного) свободными членами:


Пример 2.  Решить систему линейных уравнений методом Крамера:

.

Решение. Находим определитель системы:

Следовательно, система является определённой. Для нахождения её решения вычисляем определители

По формулам Крамера находим:

Итак, (1; 0; -1) – единственное решение системы.

Для проверки решений систем уравнений 3 Х 3 и 4 Х 4 можно воспользоваться онлайн-калькулятором, решающим методом Крамера.

Если в системе линейных уравнений в одном или нескольких уравнениях отсутствуют какие-либо переменные, то в определителе соответствующие им элементы равны нулю! Таков следующий пример.

Пример 3.  Решить систему линейных уравнений методом Крамера:

.

Решение. Находим определитель системы:

Посмотрите внимательно на систему уравнений и на определитель системы и повторите ответ на вопрос, в каких случаях один или несколько элементов определителя равны нулю. Итак, определитель не равен нулю, следовательно, система является определённой. Для нахождения её решения вычисляем определители при неизвестных

По формулам Крамера находим:

Итак, решение системы – (2; -1; 1).

Для проверки решений систем уравнений 3 Х 3 и 4 Х 4 можно воспользоваться онлайн-калькулятором, решающим методом Крамера.

Как уже говорилось, если определитель системы равен нулю, а определители при неизвестных не равны нулю, система несовместна, то есть решений не имеет. Проиллюстрируем следующим примером.

Пример 6. Решить систему линейных уравнений методом Крамера:

Решение. Находим определитель системы:

Определитель системы равен нулю, следовательно, система линейных уравнений либо несовместна и определённа, либо несовместна, то есть не имеет решений. Для уточнения вычисляем определители при неизвестных

Определители при неизвестных не равны нулю, следовательно, система несовместна, то есть не имеет решений.

Для проверки решений систем уравнений 3 Х 3 и 4 Х 4 можно воспользоваться онлайн-калькулятором, решающим методом Крамера.

В задачах на системы линейных уравнений встречаются и такие, где кроме букв, обозначающих переменные, есть ещё и другие буквы.

Эти буквы обозначают некоторое число, чаще всего действительное. На практике к таким уравнениям и системам уравнений приводят задачи на поиск общих свойств каких-либо явлений и предметов. То есть, изобрели вы какой-либо новый материал или устройство, а для описания его свойств, общих независимо от величины или количества экземпляра, нужно решить систему линейных уравнений, где вместо некоторых коэффициентов при переменных – буквы. За примерами далеко ходить не надо.

Пример 7. Решить систему линейных уравнений методом Крамера:

Здесь a – некоторое вещественное число. Решение. Находим определитель системы:

Находим определители при неизвестных

По формулам Крамера находим:

,

.

Следующий пример – на аналогичную задачу, только увеличивается количество уравнений, переменных, и букв, обозначающих некоторое действительное число.

Пример 8.  Решить систему линейных уравнений методом Крамера:

Решение. Находим определитель системы:

Находим определители при неизвестных

По формулам Крамера находим:

,

,

.

И, наконец, система четырёх уравнений с четырьмя неизвестными.

Пример 9. Решить систему линейных уравнений методом Крамера:

.

Внимание! Методы вычисления определителей четвёртого порядка здесь объясняться не будут. За этим – на соответствующий раздел сайта. Но небольшие комментарии будут. Решение. Находим определитель системы:

Небольшой комментарий. В первоначальном определителе из элементов второй строки были вычтены элементы четвёртой строки, из элементов третьей строки – элементы четвёртой строки, умноженной на 2, из элементов четвёртой строки – элементы первой строки, умноженной на 2. Преобразования первоначальных определителей при трёх первых неизвестных произведены по такой же схеме. Находим определители при неизвестных

Для преобразований определителя при четвёртом неизвестном из элементов первой строки были вычтены элементы четвёртой строки.

По формулам Крамера находим:

,

,

,

.

Итак, решение системы – (1; 1; -1; -1).

Для проверки решений систем уравнений 3 Х 3 и 4 Х 4 можно воспользоваться онлайн-калькулятором, решающим методом Крамера.

Самые внимательные, наверное, заметили, что в статье не было примеров решения неопределённых систем линейных уравнений. А всё потому, что методом Крамера решить такие системы невозможно, можно лишь констатировать, что система неопределённа. Решения таких систем даёт метод Гаусса.

Другое по теме “Системы уравнений и неравенств”

Начало темы “Линейная алгебра”

Поделиться с друзьями

Решение систем линейных уравнений методом Крамера онлайн

Одним из способов решения системы линейных алгебраических уравнений (СЛАУ) является метод Крамера . Предположим, нам дана СЛАУ вида:

a11x1a12x2a13x3b1a21x1a22x2a23x3b2a31x1a32x2a33x3b3

Требуется её решить, т.е. найти такие значения переменных x1, x2, x3 чтобы при подстановке их в исходную СЛАУ, последняя обращалась в верное тождество. Чтобы проиллюстрировать метод Крамера, запишем исходную СЛАУ в матричной форме:

a11a12a13a21a22a23a31a32a33x1x2x3b1b2b3

Первым шагом метода Крамера, является нахождение определителя матрицы СЛАУ:

Δa11a12a13a21a22a23a31a32a33

Если полученный определитель отличен от нуля, тогда исходная СЛАУ имеет единственное решение, которое может быть найдено методом Крамера. Если полученный определитель равен нулю, тогда исходная СЛАУ может не иметь решений или иметь бесконечное множество решений которые не могут быть найдены методом Крамера.

Предположим, что полученный определитель не равен нулю:

Δ0

тогда, по методу Крамера, решения находятся по формулам:

xΔxΔyΔyΔzΔzΔ

причем ∆x, ∆y и ∆z – определители полученные из определителя ∆ путем замены соответствующего столбца на вектор свободных коэффициентов. Например, определитель ∆x получается из определителя ∆ путем замены 1-ого столбца на вектор свободных коэффициентов:

Δxb1a12a13b2a22a23b3a32a33

аналогичным образом нужно сформировать определители ∆y и ∆z. Стоит отметить, что метод Крамера применим к СЛАУ в которых число уравнений равно числу неизвестных.

Данный онлайн калькулятор решает СЛАУ методом Крамера с описанием пошагового хода решения на русском языке. Коэффициенты СЛАУ могут быть не только числами или дробями, но также и параметрами. Для работы калькулятора необходимо ввести уравнения и выбрать переменные СЛАУ, которые необходимо найти.

калькулятор крамер

Вы искали калькулятор крамер? На нашем сайте вы можете получить ответ на любой математический вопрос здесь. Подробное решение с описанием и пояснениями поможет вам разобраться даже с самой сложной задачей и калькулятор крамера, не исключение. Мы поможем вам подготовиться к домашним работам, контрольным, олимпиадам, а так же к поступлению в вуз. И какой бы пример, какой бы запрос по математике вы не ввели – у нас уже есть решение. Например, «калькулятор крамер».

Применение различных математических задач, калькуляторов, уравнений и функций широко распространено в нашей жизни. Они используются во многих расчетах, строительстве сооружений и даже спорте. Математику человек использовал еще в древности и с тех пор их применение только возрастает. Однако сейчас наука не стоит на месте и мы можем наслаждаться плодами ее деятельности, такими, например, как онлайн-калькулятор, который может решить задачи, такие, как калькулятор крамер,калькулятор крамера,калькулятор крамера матрицы онлайн,калькулятор крамера онлайн,калькулятор крамера с решением,калькулятор матриц крамера,калькулятор матриц метод крамера,калькулятор матриц метод крамера онлайн,калькулятор матриц методом крамера,калькулятор матриц онлайн метод крамера,калькулятор матриц онлайн методом крамера,калькулятор матриц онлайн с решением метод крамера,калькулятор матриц онлайн с решением методом крамера,калькулятор матриц с решением методом крамера онлайн,калькулятор матрица метод крамера,калькулятор матрицы крамера онлайн,калькулятор матрицы метод крамера,калькулятор матрицы метод крамера онлайн,калькулятор матрицы методом крамера,калькулятор матрицы онлайн метод крамера,калькулятор матрицы онлайн методом крамера,калькулятор матрицы онлайн с решением метод крамера,калькулятор матричный метод крамера,калькулятор метод крамера,калькулятор метод крамера онлайн с решением,калькулятор метода крамера,калькулятор методом крамера,калькулятор методом крамера онлайн,калькулятор онлайн для метода крамера,калькулятор онлайн матриц метод крамера,калькулятор онлайн матрицы метод крамера,калькулятор онлайн матрицы методом крамера,калькулятор онлайн методом крамера,калькулятор онлайн по формуле крамера онлайн калькулятор,калькулятор онлайн правило крамера,калькулятор онлайн решение матриц методом крамера,калькулятор онлайн решение систем методом крамера,калькулятор по методу крамера,калькулятор по формуле крамера,калькулятор решение методом крамера,калькулятор решение систем линейных уравнений методом крамера,калькулятор решение систем методом крамера,калькулятор решение уравнений методом крамера,калькулятор решения методом крамера,калькулятор системы уравнений онлайн методом крамера онлайн,калькулятор слау методом крамера,калькулятор формула крамера,калькулятор формула крамера онлайн,крамер калькулятор,крамер калькулятор онлайн,крамер онлайн,крамер онлайн калькулятор,крамера онлайн,крамера онлайн калькулятор,крамера решение онлайн,линейное уравнение методом крамера онлайн,матрица калькулятор метод крамера,матрица калькулятор онлайн метод крамера,матрица крамера онлайн,матрица метод крамера калькулятор,матрица метод крамера онлайн,матрица метод крамера онлайн калькулятор,матрица методом крамера онлайн,матрица онлайн калькулятор метод крамера,матрица онлайн крамера,матрица онлайн метод крамера,матрица онлайн методом крамера,матрицы калькулятор метод крамера,матрицы калькулятор методом крамера,матрицы калькулятор онлайн крамера,матрицы калькулятор онлайн метод крамера,матрицы метод крамера калькулятор,матрицы метод крамера онлайн,матрицы метод крамера онлайн калькулятор,матрицы метод крамера онлайн калькулятор с подробным решением,матрицы методом крамера калькулятор,матрицы онлайн калькулятор крамера,матрицы онлайн калькулятор метод крамера,матрицы онлайн калькулятор методом крамера,матрицы онлайн калькулятор с подробным решением метод крамера,матрицы онлайн калькулятор с решением метод крамера,матрицы онлайн метод крамера,матрицы онлайн методом крамера онлайн,матрицы онлайн решение методом крамера,матрицы онлайн решение методом крамера онлайн с,матрицы решение методом крамера онлайн,матрицы решение методом крамера онлайн с решением,матрицы решение онлайн методом крамера,матрицы решение онлайн методом крамера онлайн с,матричный калькулятор метод крамера,метод крамера калькулятор,метод крамера калькулятор онлайн,метод крамера калькулятор с подробным решением,метод крамера матриц онлайн калькулятор,метод крамера матрица онлайн,метод крамера матрицы калькулятор,метод крамера матрицы онлайн,метод крамера матрицы онлайн калькулятор,метод крамера матричный калькулятор,метод крамера онлайн,метод крамера онлайн калькулятор,метод крамера онлайн калькулятор матрицы,метод крамера онлайн калькулятор с подробным решением,метод крамера онлайн калькулятор с подробным решением и с проверкой,метод крамера онлайн калькулятор с подробным решением матрицы,метод крамера онлайн калькулятор с решением,метод крамера онлайн решение,метод крамера онлайн решение матриц,метод крамера онлайн с решением,метод крамера примеры с решением калькулятор,метод крамера примеры с решением онлайн,метод крамера решение матриц онлайн,метод крамера решение онлайн,метод крамера решения систем линейных уравнений онлайн,метод крамера с решением онлайн,метод решение матриц крамера онлайн,метода крамера калькулятор,методом крамера калькулятор,методом крамера онлайн,методом крамера онлайн калькулятор,методом крамера решить онлайн,методом крамера решить уравнение онлайн,онлайн калькулятор для метода крамера,онлайн калькулятор крамер,онлайн калькулятор крамера,онлайн калькулятор линейных уравнений методом крамера онлайн,онлайн калькулятор матриц метод крамера,онлайн калькулятор матриц метод крамера с решением,онлайн калькулятор матриц методом крамера,онлайн калькулятор матриц с решением метод крамера,онлайн калькулятор матрицы крамера,онлайн калькулятор матрицы метод крамера,онлайн калькулятор матрицы методом крамера,онлайн калькулятор метод крамера,онлайн калькулятор метод крамера матрицы,онлайн калькулятор метод крамера с решением,онлайн калькулятор методом крамера,онлайн калькулятор правило крамера,онлайн калькулятор решение линейных уравнений методом крамера,онлайн калькулятор решение матриц методом крамера,онлайн калькулятор решение методом крамера,онлайн калькулятор решение систем линейных уравнений методом крамера,онлайн калькулятор решение систем методом крамера,онлайн калькулятор решение систем уравнений методом крамера,онлайн калькулятор решение системы уравнений методом крамера,онлайн калькулятор решить систему методом крамера,онлайн калькулятор решить систему по правилу крамера,онлайн калькулятор систем линейных уравнений методом крамера онлайн,онлайн калькулятор систем методом крамера онлайн,онлайн калькулятор систем уравнений методом крамера онлайн,онлайн калькулятор система уравнений методом крамера онлайн,онлайн калькулятор системы уравнений методом крамера онлайн,онлайн калькулятор формула крамера,онлайн калькулятор формулы крамера,онлайн крамера,онлайн матрица крамера,онлайн матрица метод крамера,онлайн матрица методом крамера,онлайн матрицы метод крамера,онлайн методом крамера,онлайн правило крамера,онлайн решение крамера,онлайн решение крамера метод,онлайн решение линейных уравнений методом крамера,онлайн решение матриц крамера,онлайн решение матриц метод крамера,онлайн решение матрицы методом крамера,онлайн решение метод крамера,онлайн решение методом крамера,онлайн решение по крамеру,онлайн решение по формулам крамера онлайн,онлайн решение по формуле крамера,онлайн решение систем линейных уравнений методом крамера,онлайн решение систем линейных уравнений методом крамера онлайн,онлайн решение систем методом крамера,онлайн решение систем методом крамера онлайн с подробным решением,онлайн решение систем уравнений методом крамера,онлайн решение системы линейных уравнений методом крамера,онлайн решение системы линейных уравнений методом крамера онлайн,онлайн решение системы методом крамера,онлайн решение системы методом крамера онлайн калькулятор,онлайн решение системы уравнений методом крамера,онлайн решение слау методом крамера,онлайн решение уравнений крамера,онлайн решение уравнений методом крамера,онлайн решение уравнения методом крамера,онлайн решить систему линейных уравнений методом крамера,онлайн система крамера,онлайн система линейных уравнений методом крамера онлайн,онлайн система уравнений методом крамера,онлайн формула крамера,определитель онлайн методом крамера онлайн,по крамеру решение онлайн,по формулам крамера решить систему онлайн,по формуле крамера калькулятор,по формуле крамера решить систему уравнений онлайн,правило крамера калькулятор онлайн,правило крамера онлайн,правило крамера онлайн калькулятор,решение крамера онлайн,решение линейных систем уравнений методом крамера онлайн,решение линейных уравнений методом крамера калькулятор онлайн,решение линейных уравнений методом крамера онлайн,решение линейных уравнений методом крамера онлайн калькулятор,решение линейных уравнений онлайн методом крамера,решение матриц крамера онлайн,решение матриц метод крамера онлайн,решение матриц методом крамера онлайн,решение матриц методом крамера онлайн калькулятор,решение матриц методом крамера онлайн с подробным решением,решение матриц онлайн калькулятор методом крамера,решение матриц онлайн крамера,решение матриц онлайн метод крамера,решение матриц онлайн метод крамера онлайн,решение матриц онлайн методом крамера,решение матриц онлайн методом крамера онлайн,решение матриц онлайн методом крамера с подробным решением,решение матриц онлайн с подробным решением методом крамера,решение матрицы крамера онлайн,решение матрицы методом крамера онлайн,решение матрицы методом крамера онлайн с подробным решением,решение матрицы методом крамера онлайн с решением,решение матрицы онлайн крамера,решение матрицы онлайн методом крамера,решение матрицы онлайн методом крамера онлайн,решение матрицы онлайн методом крамера онлайн с,решение матрицы онлайн с подробным решением методом крамера,решение матричных уравнений методом крамера онлайн,решение матричных уравнений онлайн методом крамера,решение метод крамера онлайн,решение метод крамера решение онлайн,решение методом крамера калькулятор,решение методом крамера калькулятор онлайн,решение методом крамера матрицы онлайн,решение методом крамера матрицы онлайн калькулятор,решение методом крамера онлайн,решение методом крамера онлайн калькулятор,решение методом крамера онлайн с подробным решением,решение онлайн крамера,решение онлайн линейных уравнений методом крамера,решение онлайн матриц метод крамера,решение онлайн матрицы методом крамера,решение онлайн метод крамера,решение онлайн метод крамера онлайн,решение онлайн методом крамера,решение онлайн по крамеру,решение онлайн по формуле крамера,решение онлайн систем линейных уравнений методом крамера,решение онлайн систем линейных уравнений методом крамера онлайн,решение онлайн систем методом крамера,решение онлайн систем методом крамера онлайн с подробным решением,решение онлайн системы методом крамера онлайн калькулятор,решение онлайн слау методом крамера,решение по крамеру онлайн,решение по методу крамера онлайн,решение по формуле крамера онлайн,решение систем линейных уравнений методом крамера калькулятор,решение систем линейных уравнений методом крамера калькулятор онлайн,решение систем линейных уравнений методом крамера онлайн,решение систем линейных уравнений методом крамера онлайн калькулятор,решение систем линейных уравнений методом крамера онлайн с решением,решение систем линейных уравнений онлайн методом крамера,решение систем линейных уравнений онлайн методом крамера онлайн,решение систем линейных уравнений онлайн методом крамера онлайн с,решение систем методом крамера калькулятор,решение систем методом крамера калькулятор онлайн,решение систем методом крамера онлайн,решение систем методом крамера онлайн калькулятор,решение систем онлайн методом крамера,решение систем онлайн методом крамера онлайн с подробным решением,решение систем уравнений метод крамера онлайн,решение систем уравнений методом крамера онлайн,решение систем уравнений методом крамера онлайн калькулятор,решение систем уравнений методом крамера онлайн с подробным решением,решение систем уравнений онлайн метод крамера,решение систем уравнений онлайн методом крамера,решение систем уравнений онлайн методом крамера онлайн,решение систем уравнений онлайн с подробным решением методом крамера,решение системы линейных уравнений методом крамера онлайн,решение системы линейных уравнений методом крамера онлайн с решением,решение системы линейных уравнений онлайн методом крамера,решение системы линейных уравнений онлайн методом крамера онлайн,решение системы линейных уравнений онлайн методом крамера с решением,решение системы методом крамера онлайн,решение системы онлайн методом крамера,решение системы уравнений методом крамера онлайн,решение системы уравнений методом крамера онлайн калькулятор,решение системы уравнений методом крамера онлайн с решением,решение системы уравнений онлайн методом крамера,решение слау методом крамера онлайн,решение слау методом крамера онлайн с решением,решение слау онлайн методом крамера,решение уравнений крамера онлайн,решение уравнений методом крамера калькулятор,решение уравнений методом крамера онлайн,решение уравнений онлайн крамера,решение уравнений онлайн методом крамера,решение уравнений онлайн по формуле крамера онлайн,решение уравнения методом крамера онлайн,решение уравнения онлайн методом крамера,решить линейное уравнение методом крамера онлайн,решить линейное уравнение онлайн методом крамера,решить матрицу методом крамера онлайн,решить матрицу методом крамера онлайн калькулятор,решить матрицу методом крамера онлайн с подробным решением,решить матрицу онлайн калькулятор методом крамера,решить матрицу онлайн методом крамера,решить матрицу онлайн методом крамера онлайн,решить методом крамера онлайн,решить методом крамера систему линейных уравнений онлайн,решить онлайн матрицу методом крамера,решить онлайн систему линейных уравнений методом крамера,решить онлайн слау методом крамера,решить онлайн уравнение методом крамера,решить онлайн уравнение методом крамера с подробным решением,решить по формуле крамера онлайн,решить систему линейных уравнений методом крамера онлайн,решить систему линейных уравнений методом крамера онлайн с решением,решить систему линейных уравнений методом крамера решить онлайн,решить систему линейных уравнений онлайн методом крамера,решить систему линейных уравнений онлайн по формулам крамера,решить систему линейных уравнений по формулам крамера онлайн,решить систему методом крамера онлайн,решить систему методом крамера онлайн калькулятор,решить систему методом крамера онлайн с подробным решением,решить систему онлайн методом крамера,решить систему по правилу крамера онлайн,решить систему по правилу крамера онлайн калькулятор,решить систему по формулам крамера онлайн,решить систему по формулам крамера онлайн калькулятор,решить систему уравнений методом крамера онлайн,решить систему уравнений методом крамера онлайн с подробным решением,решить систему уравнений онлайн методом крамера,решить систему уравнений онлайн по формуле крамера,решить систему уравнений по формулам крамера онлайн с решением,решить систему уравнений по формуле крамера онлайн,решить слау методом крамера онлайн,решить слау онлайн методом крамера,решить уравнение методом крамера онлайн,решить уравнение методом крамера онлайн с подробным решением,решить уравнение онлайн методом крамера,система линейных уравнений методом крамера онлайн,система линейных уравнений методом крамера онлайн калькулятор,система линейных уравнений онлайн калькулятор методом крамера,система линейных уравнений онлайн методом крамера,система линейных уравнений онлайн методом крамера онлайн,система методом крамера онлайн,система уравнений методом крамера онлайн,система уравнений онлайн методом крамера,систему линейных уравнений решить по формулам крамера онлайн,слау методом крамера калькулятор,слау методом крамера онлайн,слау методом крамера онлайн решение,слау онлайн методом крамера онлайн,слау решение методом крамера онлайн,спецкласс ру онлайн калькулятор,формула крамера для решения системы линейных уравнений онлайн,формула крамера калькулятор,формула крамера онлайн,формула крамера онлайн калькулятор,формулы крамера онлайн калькулятор. На этой странице вы найдёте калькулятор, который поможет решить любой вопрос, в том числе и калькулятор крамер. Просто введите задачу в окошко и нажмите «решить» здесь (например, калькулятор крамера матрицы онлайн).

Где можно решить любую задачу по математике, а так же калькулятор крамер Онлайн?

Решить задачу калькулятор крамер вы можете на нашем сайте https://pocketteacher.ru. Бесплатный онлайн решатель позволит решить онлайн задачу любой сложности за считанные секунды. Все, что вам необходимо сделать – это просто ввести свои данные в решателе. Так же вы можете посмотреть видео инструкцию и узнать, как правильно ввести вашу задачу на нашем сайте. А если у вас остались вопросы, то вы можете задать их в чате снизу слева на странице калькулятора.

Метод Крамера

Метод Крамера предназначен для решения тех систем линейных алгебраических уравнений (СЛАУ), у которых определитель матрицы системы отличен от нуля. Естественно, при этом подразумевается, что матрица системы квадратна (понятие определителя существует только для квадратных матриц). Решение системы уравнений методом Крамера проходит за три шага простого алгоритма:

  1. Составить определитель матрицы системы (его называют также определителем системы), и убедиться, что он не равен нулю, т.е. $\Delta\neq 0$.
  2. Для каждой переменной $x_i$($i=\overline{1,n}$) необходимо составить определитель $\Delta_{x_i}$, полученный из определителя $\Delta$ заменой i-го столбца столбцом свободных членов заданной СЛАУ.
  3. Найти значения неизвестных по формуле $x_i=\frac{\Delta_{x_{i}}}{\Delta}$ ($i=\overline{1,n}$).

Перед переходом к чтению примеров рекомендую ознакомиться с правилами вычисления определителей второго и третьего порядка, изложенными здесь.

Пример №1

Решить СЛАУ $\left\{\begin{aligned} & 3x_1+2x_2=-11;\\ & -x_1+5x_2=15. \end{aligned}\right. $ методом Крамера.

Решение

Матрица системы такова: $ A=\left( \begin{array} {cc} 3 & 2\\ -1 & 5 \end{array} \right)$. Определитель этой матрицы:

$$\Delta=\left| \begin{array} {cc} 3 & 2\\ -1 & 5 \end{array}\right|=3\cdot 5-2\cdot(-1)=17.$$

Как вычисляется определитель второго порядка можете глянуть здесь.

Так как определитель системы не равен нулю, то продолжаем решение методом Крамера. Вычислим значения двух определителей: $\Delta_{x_1}$ и $\Delta_{x_2}$. Определитель $\Delta_{x_1}$ получаем из определителя $\Delta=\left| \begin{array} {cc} 3 & 2\\ -1 & 5 \end{array}\right|$ заменой первого столбца (именно этот столбец содержит коэффициенты при $x_1$) столбцом свободных членов $\left(\begin{array} {c} -11\\ 15\end{array}\right)$:

$$ \Delta_{x_1}=\left|\begin{array}{cc}-11&2\\15&5\end{array}\right|=-55-30=-85. $$

Аналогично, заменяя второй столбец в $\Delta=\left|\begin{array}{cc}3&2\\-1&5\end{array}\right|$ столбцом свободных членов, получим:

$$ \Delta_{x_2}=\left|\begin{array} {cc} 3 & -11\\ -1 & 15\end{array}\right|=45-11=34. $$

Теперь можно найти значения неизвестных $x_1$ и $x_2$.

$$x_1=\frac{\Delta_{x_1}}{\Delta}=\frac{-85}{17}=-5;\;x_2=\frac{\Delta_{x_2}}{\Delta}=\frac{34}{17}=2.$$

В принципе, можно ещё проверить, правильно ли решена система методом Крамера. Подставим в заданную СЛАУ $x_1=-5$, $x_2=2$:

$$\left\{\begin{aligned} & 3x_1+2x_2=3\cdot(-5)+2\cdot{2}=-11;\\ & -x_1+5x_2=-(-5)+5\cdot{2}=15. \end{aligned}\right.$$

Проверка пройдена, решение системы уравнений методом Крамера найдено верно. Осталось лишь записать ответ.

Ответ: $x_1=-5$, $x_2=2$.

Пример №2

Решить СЛАУ $ \left\{\begin{aligned} & 2x_1+x_2-x_3=3;\\ & 3x_1+2x_2+2x_3=-7;\\ & x_1+x_3=-2. \end{aligned} \right.$, используя метод Крамера.

Решение

Определитель системы:

$$\Delta=\left| \begin{array} {ccc} 2 & 1 & -1\\ 3 & 2 & 2 \\ 1 & 0 & 1 \end{array}\right|=4+2+2-3=5. $$

Как вычисляется определитель третьего порядка можете глянуть здесь.

Заменяя первый столбец в $\Delta$ столбцом свободных членов, получим $\Delta_{x_1}$:

$$ \Delta_{x_1}=\left| \begin{array} {ccc} 3 & 1 & -1\\ -7 & 2 & 2 \\ -2 & 0 & 1 \end{array}\right|=6-4-4+7=5. $$

Заменяя второй столбец в $\Delta$ столбцом свободных членов, получим $\Delta_{x_2}$:

$$ \Delta_{x_2}=\left| \begin{array} {ccc} 2 & 3 & -1\\ 3 & -7 & 2 \\ 1 & -2 & 1 \end{array}\right|=-14+6+6-7-9+8=-10. $$

Заменяя третий столбец в $\Delta$ столбцом свободных членов, получим $\Delta_{x_3}$:

$$ \Delta_{x_3}=\left| \begin{array} {ccc} 2 & 1 & 3\\ 3 & 2 & -7 \\ 1 & 0 & -2 \end{array}\right|=-8-7-6+6=-15. $$

Учитывая все вышеизложенное, имеем:

$$ x_1=\frac{\Delta_{x_1}}{\Delta}=\frac{5}{5}=1;\; x_2=\frac{\Delta_{x_2}}{\Delta}=\frac{-10}{5}=-2; \; x_3=\frac{\Delta_{x_3}}{\Delta}=\frac{-15}{5}=-3. $$

Метод Крамера завершён. Можно проверить, верно ли решена система уравнений методом Крамера, подставив значения $x_1=1$, $x_2=-2$ и $x_3=-3$ в заданную СЛАУ:

$$\left\{\begin{aligned} & 2x_1+x_2-x_3=2\cdot{1}+(-2)-(-3)=3;\\ & 3x_1+2x_2+2x_3=3\cdot{1}+2\cdot(-2)+2\cdot(-3)=-7;\\ & x_1+x_3=1+(-3)=-2. \end{aligned} \right.$$

Проверка пройдена, решение системы уравнений методом Крамера найдено верно.

Ответ: $x_1=1$, $x_2=-2$, $x_3=-3$.

Пример №3

Решить СЛАУ $\left\{\begin{aligned} & 2x_1+3x_2-x_3=15;\\ & -9x_1-2x_2+5x_3=-7. \end{aligned}\right.$ используя метод Крамера.

Решение

Матрица системы $ \left( \begin{array} {ccc} 2 & 3 & -1\\ -9 & -2 & 5 \end{array} \right) $ не является квадратной. Однако это вовсе не означает, что решение системы уравнений методом Крамера невозможно. Преобразуем заданную СЛАУ, перенеся переменную $x_3$ в правые части уравнений:

$$ \left \{ \begin{aligned} & 2x_1+3x_2=x_3+15;\\ & -9x_1-2x_2=-5x_3-7. \end{aligned} \right. $$

Теперь матрица системы $ \left( \begin{array} {cc} 2 & 3 \\ -9 & -2 \end{array} \right) $ стала квадратной, и определитель её $\Delta=\left| \begin{array} {cc} 2 & 3\\ -9 & -2 \end{array}\right|=-4+27=23$ не равен нулю. Применим метод Крамера аналогично предыдущим примерам:

Ответ можно записать в таком виде: $\left\{\begin{aligned} & x_1=\frac{13x_3-9}{23};\\ & x_2=\frac{-x_3+121}{23};\\ & x_3\in R. \end{aligned}\right.$ Переменные $x_1$, $x_2$ – базисные (в иной терминологии – основные), а переменная $x_3$ – свободная (в иной терминологии – неосновная). Проверка, при необходимости, проводится так же, как и в предыдущих примерах.

Примечание

В подобных примерах возможна ситуация, когда после переноса переменной (или переменных) в правые части уравнений, определитель системы равняется нулю. В этом случае можно перенести в правую часть иную переменную (или переменные). Например, рассмотрим СЛАУ $\left\{\begin{aligned} & 2x_1-5x_2+10x_3=14;\\ & -4x_1+10x_2-7x_3=5. \end{aligned}\right.$. Если перенести в правые части уравнений $x_3$, получим: $ \left\{\begin{aligned} &2x_1-5x_2=-10x_3+14;\\ &-4x_1+10x_2=7x_3+5. \end{aligned}\right.$. Определитель данной системы $\Delta=\left| \begin{array} {cc} 2 & -5\\ -4 & 10 \end{array}\right|=20-20=0$. Однако если перенести в правые части уравнений переменную $x_2$, то получим систему $ \left\{\begin{aligned} &2x_1+10x_3=5x_2+14;\\ &-4x_1-7x_3=-10x_2+5. \end{aligned}\right.$, определитель которой $\Delta=\left| \begin{array} {cc} 2 & 10\\ -4 & -7 \end{array}\right|=-14+40=26$ не равен нулю. Дальнейшее решение аналогично рассмотренному в примере №3.

Пример №4

Решить СЛАУ

$$\left\{\begin{aligned} &x_1-5x_2-x_3-2x_4+3x_5=0;\\ &2x_1-6x_2+x_3-4x_4-2x_5=0; \\ &-x_1+4x_2+5x_3-3x_4=0. \end{aligned}\right.$$

методом Крамера.

Решение

Матрица системы $\left(\begin{array} {ccccc} 1 & -5 & -1 & -2 & 3 \\ 2 & -6 & 1 & -4 & -2 \\ -1 & 4 & 5 & -3 & 0 \end{array}\right)$ не является квадратной. Преобразуем заданную СЛАУ, перенеся переменные $x_4$, $x_5$ в правые части уравнений, и применим метод Крамера:

$$ \left\{\begin{aligned} & x_1-5x_2-x_3=2x_4-3x_5;\\ & 2x_1-6x_2+x_3=4x_4+2x_5; \\ & -x_1+4x_2+5x_3=3x_4. \end{aligned}\right.$$

Ответ таков: $\left\{\begin{aligned} & x_1=\frac{-17x_4+144x_5}{19};\\ & x_2=\frac{-15x_4+41x_5}{19};\\ & x_3=\frac{20x_4-4x_5}{19}; \\ & x_4\in R; \; x_5\in R. \end{aligned}\right.$ Переменные $x_1$, $x_2$, $x_3$ – базисные, переменные $x_4$, $x_5$ – свободные.

Естественно, что применение метода Крамера в случаях вроде того, что рассмотрен в примере №4, не всегда оправдано с точки зрения временных затрат. Мы ведь не можем гарантировать, что после переноса каких-либо переменных в правые части уравнений, определитель системы не будет равен нулю. А перебирать различные варианты – слишком долгий процесс. Гораздо удобнее в таком случае применить метод Гаусса. Я привёл пример №4 лишь с одной целью – показать, что метод Крамера применим вне зависимости от содержимого правых частей уравнений заданной СЛАУ (числа, переменные, функции – не имеет значения). Главное, чтобы определитель матрицы системы был отличен от нуля.

Решение систем линейных уравнений методом Крамера

Краткая теория


Рассмотрим частный случай системы линейных уравнений, когда число уравнений системы совпадает с числом неизвестных, подлежащих нахождению, то есть когда .

Пусть дана система  линейных уравнений с  неизвестными:

Определитель,  составленный из коэффициентов  называется определителем системы:

Рассмотрим случай, когда  – в этом случае система является определенной, то есть имеет единственное решение.

Напишем определитель, который будет отличаться от определителя  только -м столбцом, который заменен столбцом свободных членов. Обозначив этот определитель через , будем иметь:

Систему уравнений можно решить по следующим формулам, придавая индексу  значения :

Рассмотренный метод решения системы уравнений называется методом Крамера, а формулы – формулами Крамера.

Другие методы решения системы линейных алгебраических уравнений (СЛАУ):

Пример решения задачи


Задача

Решить систему линейных алгебраических уравнений (СЛАУ) методом Крамера. 

Решение

Если вам сейчас не требуется платная помощь с решением задач, контрольных работ и типовых расчетов, но может потребоваться в дальнейшем, то, чтобы не потерять контакт
вступайте в группу ВК
сохраните контакт WhatsApp (+79688494598)
сохраните контакт Телеграм (@helptask) .

Решим систему уравнений. Для этого вычислим определители, составленные по правилам Крамера:

Ответ:

Если вам сейчас не требуется платная помощь с решением задач, контрольных работ и типовых расчетов, но может потребоваться в дальнейшем, то, чтобы не потерять контакт
вступайте в группу ВК
сохраните контакт WhatsApp (+79688494598)
сохраните контакт Телеграм (@helptask) .

На цену сильно влияет срочность решения (от суток до нескольких часов). Онлайн-помощь на экзамене/зачете (срок решения 1,5 часа и меньше) осуществляется по предварительной записи.

Заявку можно оставить прямо в чате ВКонтакте, WhatsApp или Telegram, предварительно сообщив необходимые вам сроки решения и скинув условие задач.

Как решать систему уравнений (СЛАУ) методом Крамера: примеры, описание метода

Методы Крамера и Гаусса – одни из самых популярных методов решения СЛАУ. К тому же, в ряде случаев целесообразно использовать именно конкретные методы. Сессия близка, и сейчас самое время повторить или освоить их с нуля. Сегодня разбираемся с решением методом Крамера. Ведь решение системы линейных уравнений методом Крамера – весьма полезный навык.

Системы линейных алгебраических уравнений

Система линейных алгебраических уравнений – система уравнений вида:

Набор значений x, при котором уравнения системы обращаются в тождества, называется решением системы,  a и b – вещественные коэффициенты. Простенькую систему, состоящую из двух уравнений с двумя неизвестными, можно решить в уме либо выразив одну переменную через другую. Но переменных (иксов) в СЛАУ может быть гораздо больше двух, и здесь простыми школьными манипуляциями не обойтись. Что же делать? Например, решать СЛАУ методом Крамера!

Итак, пусть система состоит из n уравнений с n неизвестными.

Такую систему можно переписать в матричном виде

Здесь A – основная матрица системы, X и B, соответственно, матрицы-столбцы неизвестных переменных и свободных членов.

Решение СЛАУ методом Крамера

Если определитель главной матрицы не равен нулю (матрица невырожденная), систему можно решать по методу Крамера.

Согласно методу Крамера, решение находится по формулам:

Здесь дельта – определитель главной матрицы, а дельта x n-ное – определитель, полученный из определителя главной матрицы путем заменой n-ного столбца на столбец свободных членов.

А теперь о том, как посчитать определитель. Например, определитель матрицы третьего порядка, который чаще всего встречается на практике, вычисляется по формуле:

В этом и заключается вся суть метода Крамера. Подставляя найденные по вышеприведенным формулам значения x в искомую систему, убеждаемся в правильности (или наоборот) нашего решения. Чтобы Вы быстрее уловили суть, приведем ниже пример подробного решения СЛАУ методом Крамера:

Даже если у Вас не получится с первого раза, не расстраивайтесь! Немного практики, и Вы начнете щелкать СЛАУ как орешки. Более того, сейчас совершенно необязательно корпеть над тетрадью, решая громоздкие выкладки и исписывая стержень. Можно легко решить СЛАУ методом Крамера в режиме онлайн, лишь подставив в готовую форму коэффициенты. Испробовать онлайн калькулятор решения методом Крамера можно, к примеру,  на этом сайте.

 

Практика – путь к успеху в решении СЛАУ

 

А если система оказалась упорной и  не сдается, Вы всегда можете обратиться за помощью к нашим авторам, например, чтобы купить конспект. Будь в системе хоть 100 неизвестных, мы обязательно решим ее верно и точно в срок!

 

Автор: Иван

Иван Колобков, известный также как Джони. Маркетолог, аналитик и копирайтер компании Zaochnik. Подающий надежды молодой писатель. Питает любовь к физике, раритетным вещам и творчеству Ч. Буковски.

Калькулятор правила Крамерса

Как найти неизвестные переменные по правилу Крамерса?

Понятие матричного определителя появилось в Германии и Японии практически в одно и то же время. Секи впервые написал об этом в 1683 году в своем «Метод решения разрозненных задач ». Секи разработал шаблон для определителей для $ 2 \ times 2 $, $ 3 \ times 3 $, Матрицы $ 4 \ times 4 $ и $ 5 \ times 5 $ и использовали их для решения уравнений. В том же году Г. Лейбниц написал о методе решения система уравнений.Этот метод хорошо известен как Правило Крамера . Определитель квадратной матрицы $ A $ – это уникальное действительное число, которое является атрибутом матрицы $ A $. Определитель матрицы $ A $ обозначается $ det (A) $ или $ | A | $.

Правило Крамера – это формула для решения системы линейных уравнений. Он выводит решение в терминах определителей матрицы и матриц, полученных из нее, путем замены одного столбца вектором-столбцом правых частей уравнений. Он назван Габриэлем Крамером (17041752), и правило для произвольного числа неизвестных опубликовано в статье [Cramer, G.{th} $ столбец основной матрицы вектором правых частей уравнений и вычисляем его определитель, $ D_x $.

  • Чтобы найти решение $ x $ системы линейных уравнений по правилу Крамера, разделите определитель $ D_x $ на главный определитель $ D $;
  • Повторите предыдущий шаг для каждой переменной;
  • Если главный определитель равен нулю, система линейных уравнений либо несовместна, либо имеет бесконечно много решений.
    Правило Крамера в двух переменных : Рассмотрим систему уравнений:
    $$ \ begin {align} & a_1x + b_1y = \ color {синий} {c_1} \\ & a_2x + b_2y = \ color {синий} {c_2} \ end {align} $$ Главный определитель $$ D = \ left | \ begin {array} {cc} a_1 и b_1 \\ a_2 и b_2 \\ \ end {массив} \ right | $$ и два других детерминанта $$ D_x = \ left | \ begin {array} {cc} \ color {blue} {c_1} & b_1 \\ \ color {синий} {c_2} & b_2 \\ \ end {массив} \ right | \ quad \ mbox {и} \ quad D_y = \ left | \ begin {array} {cc} a_1 & \ color {синий} {c_1} \\ a_2 & \ color {синий} {c_2} \\ \ end {массив} \ right | $$ С помощью определителей можно найти $ x $ и $ y $ по правилу Крамера как
    . $$ x = \ frac {D_x} {D} = \ frac {\ left | \ begin {array} {cc} \ color {blue} {c_1} & b_1 \\ \ color {синий} {c_2} & b_2 \\ \ end {массив} \ right |} {\ left | \ begin {array} {cc} a_1 и b_1 \\ a_2 & b_2 \\ \ end {массив} \ right |} \ quad \ mbox {and} \ quad y = \ frac {D_y} {D} = \ frac {\ left | \ begin {array} {cc} a_1 & \ color {синий} {c_1} \\ a_2 & \ color {синий} {c_2} \\ \ end {массив} \ right |} {\ left | \ begin {array} {cc} a_1 и b_1 \\ a_2 & b_2 \\ \ end {массив} \ right |} $$ Если каждый определитель равен нулю, система непротиворечива, а уравнения зависимы.У системы бесконечно много решений. Если $ D = 0 $ и $ D_x $ или $ D_y $ не равны нулю, система несовместима и не имеет решения.
    Правило Крамера в трех переменных : Рассмотрим систему уравнений: $$ \ begin {align} & a_1x + b_1y + c_1z = \ color {синий} {d_1} \\ & a_2x + b_2y + c_2z = \ цвет {синий} {d_2} \\ & a_3x + b_3y + c_3z = \ color {синий} {d_3} \\ \ end {align} $$ Главный определитель $$ D = \ left | \ begin {array} {ccc} a_1 и b_1 и c_1 \\ a_2 и b_2 и c_2 \\ a_3 & b_3 & c_3 \\ \ end {массив} \ right | $$ а остальные три детерминанта $$ D_x = \ left | \ begin {array} {ccc} \ color {синий} {d_1} & b_1 & c_1 \\ \ цвет {синий} {d_2} & b_2 & c_2 \\ \ color {blue} {d_3} & b_3 & c_3 \\ \ end {массив} \ right | \ quad D_y = \ left | \ begin {array} {ccc} a_1 & \ color {синий} {d_1} & c_1 \\ a_2 & \ цвет {синий} {d_2} & c_2 \\ a_3 & \ color {синий} {d_3} & c_3 \\ \ end {массив} \ right | \ quad \ mbox {and} \ quad D_z = \ left | \ begin {array} {ccc} a_1 & b_1 & \ color {синий} {d_1} \\ a_2 & b_2 & \ color {синий} {d_2} \\ a_3 & b_3 & \ color {синий} {d_3} \\ \ end {массив} \ right | $$ Решение системы трех уравнений есть $$ x = \ frac {D_x} {D}, \ quad y = \ frac {D_y} {D}, \ quad \ mbox {и} \ quad z = \ frac {D_z} {D} $$ Например, решим систему линейных уравнений: $$ \ begin {align} & 3x + 4y + 5z = 10 \\ & 5x + 6y + 7z = 12 \\ & 4x + 5y + 0z = 15 \\ \ end {align} $$ Сначала вычисляем главный определитель: $$ \ begin {align} D & = \ left | \ begin {array} {ccc} 3 и 4 и 5 \\ 5 и 6 и 7 \\ 4 и 5 и 0 \\ \ end {массив} \ right | \ & = \ left | \ begin {array} {ccc | cc} 3 и 4 и 5 и 3 и 4 \\ 5 и 6 и 7 и 5 и 6 \\ 4 и 5 и 0 и 4 и 5 \\ \ end {массив} \верно.= 3 \ cdot6 \ cdot0 + 4 \ cdot7 \ cdot4 + 5 \ cdot5 \ cdot 5-5 \ cdot6 \ cdot4-3 \ cdot7 \ cdot5-4 \ cdot6 \ cdot0 = 12 \ end {align} $$ По аналогии, $$ D_x = \ left | \ begin {array} {ccc} \ color {blue} {10} & 4 и 5 \\ \ color {blue} {12} & 6 и 7 \\ \ color {blue} {15} & 5 & 0 \\ \ end {массив} \ right | = -80, \ quad D_y = \ left | \ begin {array} {ccc} 3 & \ color {синий} {10} & 5 \\ 5 & ​​\ color {blue} {12} & 7 \\ 4 & \ color {blue} {15} & 0 \\ \ end {массив} \ right | = 100, \ quad D_z = \ left | \ begin {array} {ccc} 3 и 4 & \ color {синий} {10} \\ 5 и 6 & \ color {синий} {12} \\ 4 и 5 & \ color {синий} {15} \\ \ end {массив} \ right | = -8 $$

    Калькулятор правила Крамера – (2×2, 3×3 и 4×4) Матрицы

    Формула правила Крамера

    x = D x / D

    y = D y / D

    z = D z / D

    Где,

    D x, ​​ D y, и D z определяют матрицу x, y, и z соответственно, а

    D – определитель основной матрицы.

    Калькулятор правил Крамера эффективно решает одновременные линейные уравнения и мгновенно находит значения переменных в уравнении. Он также применяет правило Крамера для матриц 2×2, , 3×3, и 4×4, .

    Если вы знаете, как использовать правило Крамера в системе 2×2, и ищете реализацию правила Крамера в системах 3×3 или 4×4, продолжайте читать следующие разделы.

    Что такое правило Крамера?

    Правило Крамера – это метод оценки значения заданных неизвестных переменных в линейных уравнениях.Он был предложен Габриэлем Крамером в 1750 году. Используя это правило, можно легко решать совместные линейные уравнения.

    Как решить линейные уравнения с помощью правила Крамера?

    Чтобы решить совместные линейные уравнения с помощью правила Крамера, выполните следующие действия.

    Пример:

    Решите приведенные ниже уравнения для x, y, и z.

    2x + 3y + 5z = 10

    5x + 3y + 2z = 12

    x + 5y + 0z = 8

    Решение 9: 900499 Шаг 1: Используя коэффициенты, переменные и константы, создайте матрицу, как показано ниже.

    Шаг 2: Найдите определитель главной матрицы. Предположим, что основная матрица равна D.

    = 2 [(3 × 0) – (2 × 5)] – 3 [(5 × 0) – (2 × 1 )] + 5 [(5 × 5) – (3 × 1)]

    = 2 (0-10) – 3 (0-2) + 5 (25-3)

    = -20 + 6 + 110

    | D | = 96

    Шаг 3: Построение матриц x, y, и z путем замены x, y, и z столбцов основной матрицы D по постоянной матрице соответственно.

    Шаг 4: Возьмите определитель всех трех новых матриц x, y, и z .

    D x = 10 [(3 × 0) – (2 × 5)] – 3 [(12 × 0) – (2 × 8)] + 5 [(12 × 5) – (3 × 8)]

    D x = -100 + 48 + 180

    D x = 128

    D y = 2 [(12 × 0) – (2 × 8)] – 10 [(5 × 0) – (2 × 1)] + 5 [(5 × 8) – (12 × 1)]

    D y = -32 + 20 + 140

    D y = 128

    D z = 2 [(3 × 8) – (12 × 5)] – 3 [(5 × 8) – (12 × 1)] + 10 [(5 × 5) – (3 × 1)]

    D z = -72 – 84 + 220

    D z = 64

    Шаг 4: Примените правила Крамера и разместите значения.

    x = D x / D = 128/96

    x = 1,33

    y = D y / D = 128/96

    y = 1,33

    z = D z / D = 64/96

    z = 0,67

    Итак, мы получили x = 1,33, y = 1,33, и z = 0,67 после применения правила Крамера к данному уравнению 3×3 .

    Решите систему линейных уравнений по правилу Крамера онлайн

    Один из способов решения системы линейных алгебраических уравнений (СЛАУ) – использование Правило Крамера . Предположим, у нас есть СЛАУ:

    a11x1a12x2a13x3b1a21x1a22x2a23x3b2a31x1a32x2a33x3b3

    Для ее решения нужно найти такие значения переменных х 1 , х 2 , х 3 которые преобразуют исходный SLAE в правильный идентификатор.Чтобы показать, как работает правило Крамера, перепишите нашу исходную СЛАУ в матричной форме:

    a11a12a13a21a22a23a31a32a33x1x2x3b1b2b3

    Первый шаг Правило Крамера , состоит в том, чтобы проверить ценность детерминант матрицы СЛАУ:

    Δa11a12a13a21a22a23a31a32a33

    Если вычисленный определитель не равен нулю, то исходная СЛАУ имеет единственное решение, которое можно найти по правилу Крамера.Если вычисленный определитель равен нулю, то исходная СЛАУ может либо не иметь решения, либо иметь бесконечный набор решений, который не может быть найден по правилу Крамера.

    Предположим, вычисленный определитель не равен нулю:

    Δ0

    то по правилу Крамера решение СЛАУ можно найти по формулам:

    xΔxΔyΔyΔzΔzΔ

    здесь, ∆ x , ∆ y а также ∆ z являются детерминантами, производными от определителя ∆ заменив соответствующий столбец на вектор свободных коэффициентов.Например, определитель ∆ x полученный от определителя ∆ заменив первый столбец на вектор свободных коэффициентов:

    Δxb1a12a13b2a22a23b3a32a33

    Используя этот метод, можно получить определители ∆ и а также ∆ z . Следует отметить, что правило Крамера применимо к СЛАУ, в которых количество уравнений равно количеству переменных.

    Наш онлайн-калькулятор решает SLAE по правилу Крамера с пошаговым решением. Коэффициенты СЛАУ могут быть не только числами дробей, но и параметрами. Чтобы использовать калькулятор, необходимо ввести СЛАУ и выбрать переменные СЛАУ для поиска.

    Решение любых систем с помощью калькулятора правила Крамера – Онлайн калькуляторы

    Полезная информация

    Система линейных уравнений – это система цифр:
    $$ \ large {\ left \ {\ begin {array} {1} \ mathrm {a_ {11} x_ {1} + a_ {12} x_ {2}} & + & \ cdots & + & \ mathrm {a_ {1n} x_ {n} = b_ {1}}, \\ \ mathrm {a_ {21} x_ {1} + a_ {22} x_ {2}} & + & \ dotsb & + & \ mathrm {a_ {2n} x_ {n} = b_ {2}}, \\ \ vdots & \ vdots & \ ddots & \ vdots \\ \ mathrm {a_ {m1} x_ {1} + a_ {m2} x_ {2}} & + & \ cdots & + & \ mathrm {a_ {mn} x_ {n} = b_ {m}} \ end {array} \ right.} $$
    gdzie:

    \ (\ mathrm {\ large {a_ {11}, …, a_ {mn}}} \) – это коэффициенты уравнения (данных)
    \ (\ mathrm {\ large { b_ {1}, …, b_ {m}}} \) – свободные слова (данные)
    \ (\ mathrm {\ large {x_ {1}, …, x_ {n}}} \) – неизвестно

    Существует два типа систем линейных уравнений:

      • однородные системы линейных уравнений,
      • системы линейных уравнений неоднородные,

    Система линейных уравнений называется однородной, если все члены равны нулю.
    $$ \ large {\ left \ {\ begin {array} {1} \ mathrm {a_ {11} x_ {1} + a_ {12} x_ {2}} & + & \ cdots & + & \ mathrm {a_ {1n} x_ {n} = 0}, \\ \ mathrm {a_ {21} x_ {1} + a_ {22} x_ {2}} & + & \ dotsb & + & \ mathrm {a_ {2n} x_ {n} = 0}, \\ \ vdots & \ vdots & \ ddots & \ vdots \\ \ mathrm {a_ {m1} x_ {1} + a_ {m2} x_ {2}} & + & \ cdots & + & \ mathrm {a_ {mn} x_ {n} = 0} \ end {array} \ right.} $$

    Методы решения линейных уравнений делятся на два класса:

      • конечные методы – позволяют получить решение после выполнения конечного числа арифметических операций, а полученные решения обременяют только с ошибками округления.
      • итерационные методы – они заключаются в определении последовательности векторов, сходящейся к решению системы. Полученные решения допускают ошибки в методе и округлении. Однако эти методы позволяют находить решения с любой заданной точностью.

    Теорема Крамера:
    Если определитель матрицы коэффициентов системы отличен от нуля ( detA ≠ 0 ), то система линейных уравнений в векторной форме \ (\ color {blue} {x_1 } \ mathbf a_1 + \ dots + \ color {blue} {x_n} \ mathbf a_n = \ color {red} {\ mathbf b} \) имеет ровно одно решение, задаваемое формулами:
    $$ \ color {blue} {x_1 } = \ frac {\ det (\ color {red} {\ mathbf b}, \; \ mathbf a_2, \ dots, \ mathbf a_n)} {\ det (\ mathbf a_1, \ mathbf a_2, \ dots, \ mathbf a_n)}, $$ $$ \ vdots $$ $$ \ color {синий} {x_n} = \ frac {\ det (\ mathbf a_1, \ dots, \ mathbf a_ {n-1}, \; \ color {red} {\ mathbf b})} {\ det (\ mathbf a_1, \ точки, \ mathbf a_ {n-1}, \ mathbf a_n)} $$.

    so
    $$ x_i = \ frac {W_i} {W}; $$ где
      \ (W = detA \) – это определитель матрицы коэффициентов системы. Это называется главным детерминантом.
      \ (W_i \) – это определитель матрицы, которая получается из матрицы \ (A \), заменяя столбец коэффициентов неизвестного \ (\ color {blue} {x_i} \) на столбец перехватов \ (\ color {red} {b} \).

    Систему уравнений можно записать в матричной форме:
    $$ \ begin {bmatrix} a_ {11} & a_ {12} & \ dots & a_ {1n} \\ a_ {21} & a_ {22} & \ dots & a_ {2n} \\ \ vdots & \ vdots & \ vdots & \ vdots \\ a_ {m1} & a_ {m2} & \ dots & a_ {mn} \ end {bmatrix} \ begin {bmatrix} \ color {blue} {x_1} \\ \ color {blue} {x_2} \\ \ vdots \\ \ color {blue} {x_n} \ end {bmatrix} = \ begin {bmatrix} \ color {red} {b_1} \\ \ color {red} {b_2} \\ \ vdots \\ \ color {red} {b_n} \ end {bmatrix}.$$ Учитывая квадратную матрицу перехватов, где количество строк равно количеству столбцов (m = n), вычислим главный определитель W .
    $$ W = \ begin {vmatrix} a_ {11} & a_ {12} & \ dots & a_ {1n} \\ a_ {21} & a_ {22} & \ dots & a_ {2n} \\ \ vdots & \ vdots & \ vdots & \ vdots \\ a_ {m1} & a_ {m2} & \ dots & a_ {mn} \ end {vmatrix} $$ Если W ≠ 0 , мы вычисляем определители вспомогательных матриц W 1 , W 2 … W n замена соответствующего столбца коэффициентов столбцом пересечения.
    $$ W_1 = \ begin {vmatrix} \ color {red} {b_1} & a_ {12} & \ dots & a_ {1n} \\ \ color {red} {b_2} & a_ {22} & \ dots & a_ {2n} \\ \ vdots & \ vdots & \ vdots & \ vdots \\ \ color {красный} {b_n} & a_ {m2} & \ dots & a_ {mn} \ end {vmatrix}; W_2 = \ begin {vmatrix} a_ {11} & \ color {red} {b_1} & \ dots & a_ {1n} \\ a_ {21} & \ color {red} {b_2} & \ dots & a_ {2n } \\ \ vdots & \ vdots & \ vdots & \ vdots \\ a_ {m1} & \ color {красный} {b_n} & \ dots & a_ {mn} \ end {vmatrix}; …; W_n = \ begin {vmatrix} a_ {11} & a_ {12} & \ dots & \ color {red} {b_1} \\ a_ {21} & a_ {22} & \ dots & \ color {red} {b_2 } \\ \ vdots & \ vdots & \ vdots & \ vdots \\ a_ {m1} & a_ {m2} & \ dots & \ color {красный} {b_n} \ end {vmatrix} $$
    Теперь, согласно приведенной выше формуле, подставляя определители, мы можем вычислить неизвестные.

    9.8: Решение систем по правилу Крамера

    Мы узнали, как решать системы уравнений с двумя переменными и тремя переменными, а также с помощью нескольких методов: подстановки, сложения, исключения Гаусса, использования обратной матрицы и построения графиков.Некоторые из этих методов применять проще, чем другие, и они более подходят в определенных ситуациях. В этом разделе мы изучим еще две стратегии решения систем уравнений.

    Вычисление определителя матрицы 2 × 2

    Определитель – это действительное число, которое может быть очень полезно в математике, потому что у него есть несколько приложений, таких как вычисление площади, объема и других величин. Здесь мы будем использовать определители, чтобы определить, является ли матрица обратимой, используя элементы квадратной матрицы, чтобы определить, существует ли решение системы уравнений.Однако, возможно, одним из наиболее интересных приложений является их использование в криптографии. Защищенные сигналы или сообщения иногда отправляются в виде матрицы. Расшифровать данные можно только с помощью обратимой матрицы и определителя. В наших целях мы ориентируемся на определитель как на показатель обратимости матрицы. Вычисление определителя матрицы требует следования определенным шаблонам, описанным в этом разделе.

    НАЙТИ ОПРЕДЕЛЕНИЕ МАТРИЦЫ 2 × 2

    Определитель матрицы 2 × 2, учитывая

    \ (A = \ begin {bmatrix} a & b \\ c & d \ end {bmatrix} \)

    определяется как

    Обратите внимание на изменение обозначений.Есть несколько способов указать определитель, включая \ (\ det (A) \) и замену скобок в матрице прямыми линиями, \ (| A | \).

    Пример \ (\ PageIndex {1} \): поиск определителя матрицы \ (2 × 2 \)

    Найдите определитель заданной матрицы.

    \ (A = \ begin {bmatrix} 5 & 2 \\ – 6 & 3 \ end {bmatrix} \)

    Решение

    \ [\ begin {align *} \ det (A) & = \ begin {vmatrix} 5 & 2 \\ – 6 & 3 \ end {vmatrix} \\ & = 5 (3) – (- 6) (2) \\ & = 27 \ end {align *} \]

    Использование правила Крамера для решения системы двух уравнений с двумя переменными

    Теперь мы представим последний метод решения систем уравнений, использующий определители.Этот метод, известный как правило Крамера , восходит к середине 18 века и назван в честь своего новатора, швейцарского математика Габриэля Крамера (1704-1752), который представил его в 1750 году в году. Courbes algébriques . Правило Крамера – это жизнеспособный и эффективный метод поиска решений систем с произвольным числом неизвестных при условии, что у нас есть такое же количество уравнений, что и неизвестных.

    Правило Крамера даст нам единственное решение системы уравнений, если оно существует.Однако, если система не имеет решения или бесконечное количество решений, это будет обозначено нулевым определителем. Чтобы выяснить, является ли система непоследовательной или зависимой, необходимо использовать другой метод, например исключение.

    Чтобы понять правило Крамера, давайте внимательно рассмотрим, как мы решаем системы линейных уравнений с использованием основных операций со строками. Рассмотрим систему двух уравнений с двумя переменными.

    \ [\ begin {align} a_1x + b_1y & = c_1 (1) \ label {eq1} \\ a_2x + b_2y & = c_2 (2) \ label {eq2} \\ \ end {align} \]

    Мы исключаем одну переменную, используя операции со строками, и решаем для другой.Скажите, что мы хотим найти \ (x \). Если уравнение \ ref {eq2} умножается на коэффициент, противоположный коэффициенту \ (y \) в уравнении \ ref {eq1}, уравнение \ ref {eq1} умножается на коэффициент при \ (y \) в уравнении \ ref {eq2}, и мы добавляем два уравнения, переменная \ (y \) будет удалена.

    \ [\ begin {align *} & b_2a_1x + b_2b_1y = b_2c_1 & \ text {Multiply} R_1 \ text {by} b_2 \\ – & \ underline {b_1a_2x − b_1b_2y = −b_1c_2} & \ text {Multiply} R_2 \ text {by} −b_1 \\ & b_2a_1x − b_1a_2x = b_2c_1 − b_1c_2 \ end {align *} \]

    Теперь решите относительно \ (x \).

    \ [\ begin {align *} b_2a_1x − b_1a_2x & = b_2c_1 − b_1c_2 \\ x (b_2a_1 − b_1a_2) & = b_2c_1 − b_1c_2 \\ x & = \ dfrac {b_2c_1 − b_1c_2} {b_2a_1 − b_1a_2} = \ dfrac {\ begin {bmatrix} c_1 & b_1 \\ c_2 & b_2 \ end {bmatrix}} {\ begin {bmatrix} a_1 & b_1 \\ a_2 & b_2 \ end {bmatrix}} \ end {align *} \]

    Аналогично, чтобы найти \ (y \), мы исключим \ (x \).

    \ [\ begin {align *} & a_2a_1x + a_2b_1y = a_2c_1 & \ text {Multiply} R_1 \ text {by} a_2 \\ – & \ underline {a_1a_2x − a_1b_2y = −a_1c_2} & \ text {Multiply} R_2 \ текст {by} −a_1 \\ & a_2b_1y − a_1b_2y = a_2c_1 − a_1c_2 \ end {align *} \]

    Решение относительно \ (y \) дает

    \ [\ begin {align *} a_2b_1y − a_1b_2y & = a_2c_1 − a_1c_2 \\ y (a_2b_1 − a_1b_2) & = a_2c_1 − a_1c_2 \\ y & = \ dfrac {a_2c_1 − a_1c_2} {a_2b_1 − a_1b_2} = \ dfrac {a_1c_2 − a_2c_1} {a_1b_2 − a_2b_1} = \ dfrac {\ begin {bmatrix} a_1 & c_1 \\ a_2 & c_2 \ end {bmatrix}} {\ begin {bmatrix} a_1 & b_1 \\ a_2 & b_2 \ end {bmatrix}} \ end {align * } \]

    Обратите внимание, что знаменатель для \ (x \) и \ (y \) является определителем матрицы коэффициентов.

    Мы можем использовать эти формулы для решения относительно \ (x \) и \ (y \), но правило Крамера также вводит новые обозначения:

    • \ (D \): определитель матрицы коэффициентов
    • \ (D_x \): определитель числителя в решении \ (x \)

      \ [x = \ dfrac {D_x} {D} \]

    • \ (D_y \): определитель числителя в решении \ (y \)

      \ [y = \ dfrac {D_y} {D} \]

    Ключ к правилу Крамера заключается в замене интересующего столбца переменных столбцом констант и вычислении детерминантов.Тогда мы можем выразить \ (x \) и \ (y \) как частное двух определителей.

    ПРАВИЛО КРЕМЕРА ДЛЯ СИСТЕМ \ (2 × 2 \)

    Правило Крамера – это метод, который использует детерминанты для решения систем уравнений, которые имеют то же количество уравнений, что и переменные.

    Рассмотрим систему двух линейных уравнений с двумя переменными.

    \ [\ begin {align *} a_1x + b_1y & = c_1 \\ a_2x + b_2y & = c_2 \ end {align *} \]

    Решение, использующее правило Крамера, дается как

    \ [\ begin {align} x & = \ dfrac {D_x} {D} = \ dfrac {\ begin {bmatrix} c_1 & b_1 \\ c_2 & b_2 \ end {bmatrix}} {\ begin {bmatrix} a_1 & b_1 \\ a_2 & b_2 \ end { bmatrix}} \; , D \ neq 0 \\ y & = \ dfrac {D_y} {D} = \ dfrac {\ begin {bmatrix} a_1 & c_1 \\ a_2 & c_2 \ end {bmatrix}} {\ begin {bmatrix} a_1 & b_1 \\ a_2 & b_2 \ end {bmatrix }} \; , D \ neq 0 \ end {align} \]

    Если мы решаем для \ (x \), столбец \ (x \) заменяется постоянным столбцом.Если мы решаем для \ (y \), столбец \ (y \) заменяется постоянным столбцом.

    Пример \ (\ PageIndex {2} \): использование правила Крамера для решения системы \ (2 × 2 \)

    Решите следующую систему \ (2 × 2 \), используя правило Крамера.

    \ [\ begin {align *} 12x + 3y & = 15 \\ 2x-3y & = 13 \ end {align *} \]

    Решение

    Решите относительно \ (x \).

    \ [\ begin {align *} x & = \ dfrac {D_x} {D} \\ & = \ dfrac {\ begin {bmatrix} 15 & 3 \\ 13 & -3 \ end {bmatrix}} {\ begin {bmatrix} 12 & 3 \\ 2 & -3 \ end {bmatrix}} \\ & = \ dfrac {-45-39} {- 36-6} \\ & = \ dfrac {-84} {- 42} \\ & = 2 \ end {align *} \]

    Решите относительно \ (y \).

    \ [\ begin {align *} y & = \ dfrac {D_y} {D} \\ & = \ dfrac {\ begin {bmatrix} 12 & 15 \\ 2 & 13 \ end {bmatrix}} {\ begin {bmatrix} 12 & 3 \\ 2 & -3 \ end {bmatrix}} \\ & = \ dfrac {156-30} {- 36-6} \\ & = – \ dfrac {126} {42} \\ & = -3 \ end {align * } \]

    Решение: \ ((2, −3) \).

    Упражнение \ (\ PageIndex {1} \)

    Используйте правило Крамера для решения системы уравнений \ (2 × 2 \).

    \ [\ begin {align *} x + 2y & = -11 \\ -2x + y & = -13 \ end {align *} \]

    Ответ

    \ ((3, −7) \)

    Вычисление определителя матрицы 3 × 3

    Найти определитель матрицы 2 × 2 несложно, но найти определитель матрицы 3 × 3 сложнее.Один из способов – увеличить матрицу 3 × 3 повторением первых двух столбцов, получив матрицу 3 × 5. Затем мы вычисляем сумму произведений записей на по каждой из трех диагоналей (от верхнего левого угла к нижнему правому) и вычитаем произведение записей на по каждой из трех диагоналей (нижний левый верхний правый). Это легче понять с помощью наглядного пособия и примера.

    Найдите определитель матрицы 3 × 3.

    \ (A = \ begin {bmatrix} a_1 & b_1 & c_1 \\ a_2 & b_2 & c_2 \\ a_3 & b_3 & c_3 \ end {bmatrix} \)

    1. Дополнение \ (A \) с первыми двумя столбцами.

      \ (\ det (A) = \ left | \ begin {array} {ccc | cc} a_1 & b_1 & c_1 & a_1 & b_1 \\ a_2 & b_2 & c_2 & a_2 & b_2 \\ a_3 & b_3 & c_3 & a_3 & b_3 \ end {array} \ right | \)

    2. С верхнего левого угла в нижний правый: умножение значений по первой диагонали. Добавьте результат к произведению входов по второй диагонали. Добавьте этот результат к произведению записей по третьей диагонали.
    3. От левого нижнего угла до правого верхнего: вычтите произведение входов вверх по первой диагонали.Из этого результата вычтите произведение входов вверх по второй диагонали. Из этого результата вычтите произведение входов до третьей диагонали.

    Алгебра выглядит следующим образом:

    \ (| A | = a_1b_2c_3 + b_1c_2a_3 + c_1a_2b_3 − a_3b_2c_1 − b_3c_2a_1 − c_3a_2b_1 \)

    Пример \ (\ PageIndex {3} \): поиск определителя матрицы 3 × 3

    Найдите определитель матрицы \ (3 × 3 \) при

    \ (A = \ begin {bmatrix} 0 & 2 & 1 \\ 3 & −1 & 1 \\ 4 & 0 & 1 \ end {bmatrix} \)

    Решение

    Дополните матрицу первыми двумя столбцами, а затем следуйте формуле.Таким образом,

    \ [\ begin {align *} | А | & = \ left | \ begin {array} {ccc | cc} 0 & 2 & 1 & 0 & 2 \\ 3 & -1 & 1 & 3 & -1 \\ 4 & 0 & 1 & 4 & 0 \ end {array} \ right | \\ & = 0 (−1) (1) +2 (1) (4) +1 (3) (0) −4 (−1) (1) −0 (1) (0) −1 (3) (2) \\ & = 0 + 8 + 0 + 4−0−6 \\ & = 6 \ end {align *} \]

    Упражнение \ (\ PageIndex {2} \)

    Найдите определитель матрицы 3 × 3.

    \ (\ det (A) = \ begin {vmatrix} 1 & −3 & 7 \\ 1 & 1 & 1 \\ 1 & −2 & 3 \ end {vmatrix} \)

    Ответ

    \ (- 10 \)

    Q&A: Можем ли мы использовать тот же метод, чтобы найти определитель большей матрицы?

    Нет, этот метод работает только для матриц 2 × 2 и 3 × 3.Для больших матриц лучше всего использовать графическую утилиту или компьютерное программное обеспечение.

    Использование правила Крамера для решения системы трех уравнений с тремя переменными

    Теперь, когда мы можем найти определитель матрицы \ (3 × 3 \), мы можем применить правило Крамера для решения системы трех уравнений с тремя переменными. Правило Крамера простое и соответствует шаблону, соответствующему правилу Крамера для матриц \ (2 × 2 \). Однако по мере увеличения порядка матрицы до \ (3 × 3 \) требуется гораздо больше вычислений.

    Когда мы вычисляем, что детерминант равен нулю, правило Крамера не дает указания на то, что у системы нет решения или есть бесконечное количество решений. Чтобы выяснить это, мы должны выполнить устранение в системе.

    Рассмотрим систему уравнений \ (3 × 3 \).

    \ [\ begin {align} a_1x + b_1y + c_1z & = \ color {blue} d_1 \\ a_2x + b_2y + c_2z & = \ color {blue} d_2 \\ a_3x + b_3y + c_3z & = \ color {blue} d_3 \\ \ end {align} \]

    \ (x = \ dfrac {D_x} {D} \), \ (y = \ dfrac {D_y} {D} \), \ (z = \ dfrac {D_z} {D} \), \ (D ≠ 0 \)

    где

    \ [D = \ begin {vmatrix} a_1 & b_1 & c_1 \\ a_2 & b_2 & c_2 \\ a_3 & b_3 & c_3 \ end {vmatrix} \; , \; D_x = \ begin {vmatrix} \ color {blue} d_1 & b_1 & c_1 \\ \ color {blue} d_2 & b_2 & c_2 \\ \ color {blue} d_3 & b_3 & c_3 \ end {vmatrix} \; , \; D_y = \ begin {vmatrix} a_1 & \ color {blue} d_1 & c_1 \\ a_2 & \ color {blue} d_2 & c_2 \\ a_3 & \ color {blue} d_3 & c_3 \ end {vmatrix} \; , \; D_z = \ begin {vmatrix} a_1 & b_1 & \ color {blue} d_1 \\ a_2 & b_2 & \ color {blue} d_2 \\ a_3 & b_3 & \ color {blue} d_3 \ end {vmatrix} \]

    Если мы пишем определитель \ (D_x \), мы заменяем столбец \ (x \) постоянным столбцом.Если мы пишем определитель \ (D_y \), мы заменяем столбец y на столбец констант. Если мы пишем определитель \ (D_z \), мы заменяем столбец \ (z \) постоянным столбцом. Всегда проверяйте ответ.

    Пример \ (\ PageIndex {4} \): решение системы \ (3 × 3 \) с использованием правила Крамера

    Найдите решение данной системы \ (3 × 3 \), используя правило Крамера.

    \ [\ begin {align *} x + y-z & = 6 \\ 3x-2y + z & = -5 \\ x + 3y-2z & = 14 \ end {align *} \]

    Решение

    Используйте правило Крамера.

    \ (D = \ begin {vmatrix} 1 & 1 & −1 \\ 3 & −2 & 1 \\ 1 & 3 & −2 \ end {vmatrix} \), \ (D_x = \ begin {vmatrix} 6 & 1 & −1 \\ – 5 & −2 & 1 \ \ 14 & 3 & −2 \ end {vmatrix} \), \ (D_y = \ begin {vmatrix} 1 & 6 & −1 \\ 3 & −5 & 1 \\ 1 & 14 & −2 \ end {vmatrix} \), \ (D_z = \ begin {vmatrix } 1 & 1 & 6 \\ 3 & −2 & −5 \\ ​​1 & 3 & 14 \ end {vmatrix} \)

    Затем,

    \ [\ begin {align *} x & = \ dfrac {D_x} {D} & = \ dfrac {-3} {- 3} & = 1 \\ y & = \ dfrac {D_y} {D} & = \ dfrac {-9} {- 3} & = 3 \\ z & = \ dfrac {D_z} {D} & = \ dfrac {6} {- 3} & = -2 \\ \ end {align *} \]

    Решение: \ ((1,3, −2) \).

    Упражнение \ (\ PageIndex {3} \)

    Используйте правило Крамера, чтобы решить матрицу \ (3 × 3 \).

    \ [\ begin {align *} x-3y + 7z & = 13 \\ x + y + z & = 1 \\ x-2y + 3z & = 4 \ end {align *} \]

    Ответ

    \ (\ left (−2, \ dfrac {3} {5}, \ dfrac {12} {5} \ right) \)

    Пример \ (\ PageIndex {5A} \): Использование правила Крамера для решения несовместимой системы

    Решите систему уравнений, используя правило Крамера.

    \ [\ begin {align} 3x-2y & = 4 \ label {eq3} \\ 6x-4y & = 0 \ label {eq4} \ end {align} \]

    Решение

    Начнем с нахождения определителей \ (D \), \ (D_x \) и \ (D_y \).

    \ (D = \ begin {vmatrix} 3 & −2 \\ 6 & −4 \ end {vmatrix} = 3 (−4) −6 (−2) = 0 \)

    Мы знаем, что нулевой определитель означает, что либо система не имеет решения, либо имеет бесконечное число решений. Чтобы узнать, какой из них, мы используем процесс исключения. Наша цель – исключить одну из переменных.

    1. Умножьте уравнение \ ref {eq3} на \ (- 2 \).
    2. Добавьте результат в уравнение \ ref {eq4}.

    \ [\ begin {align *} & −6x + 4y = −8 \\ & \; \; \; \ underline {6x − 4y = 0} \\ & \; \; \; \; \; \ ; \; \; \; \; 0 = −8 \ end {align *} \]

    Получаем уравнение \ (0 = −8 \), которое неверно. Следовательно, у системы нет решения. График системы показывает две параллельные линии. См. Рисунок \ (\ PageIndex {1} \).

    Рисунок \ (\ PageIndex {1} \)

    Пример \ (\ PageIndex {5B} \): использование правила Крамера для решения зависимой системы

    Решите систему с бесконечным количеством решений.

    \ [\ begin {align} x-2y + 3z & = 0 \ label {eq5} \\ 3x + y-2z & = 0 \ label {eq6} \\ 2x-4y + 6z & = 0 \ label {eq7} \ end {align} \]

    Решение

    Давайте сначала найдем определитель. Создайте матрицу, дополненную первыми двумя столбцами.

    \ (\ left | \ begin {array} {ccc | cc} 1 & −2 & 3 & 1 & -2 \\ 3 & 1 & −2 & 3 & 1 \\ 2 & −4 & 6 & 2 & -4 \ end {array} \ right | \)

    Затем,

    \ (1 (1) (6) + (- 2) (- 2) (2) +3 (3) (- 4) −2 (1) (3) – (- 4) (- 2) (1 ) −6 (3) (- 2) = 0 \)

    Поскольку определитель равен нулю, решения либо нет, либо существует бесконечное количество решений.Чтобы выяснить это, нам нужно провести отбор.

    1. Умножьте уравнение \ ref {eq5} на \ (- 2 \) и добавьте результат к уравнению \ ref {eq7}:

    \ [\ begin {align *} & −2x + 4y − 6x = 0 \\ & \; \; \ underline {2x − 4y + 6z = 0} \\ & \; \; \; \; \; \ ; \; \; \; \; \; \; \; \; \; \; 0 = 0 \ end {align *} \]

    2. Получение ответа \ (0 = 0 \), утверждение, которое всегда верно, означает, что система имеет бесконечное количество решений. Изобразив систему, мы можем увидеть, что две плоскости одинаковы, и обе они пересекают третью плоскость по прямой.См. Рисунок \ (\ PageIndex {2} \).

    Рисунок \ (\ PageIndex {2} \)

    Понимание свойств детерминантов

    Есть много свойств определителей. Здесь перечислены некоторые свойства, которые могут быть полезны при вычислении определителя матрицы.

    СВОЙСТВА ДЕТЕРМИНАНТОВ

    1. Если матрица имеет верхнюю треугольную форму, определитель равен произведению элементов по главной диагонали.
    2. Когда две строки меняются местами, определитель меняет знак.{−1} \) – величина, обратная определителю матрицы \ (A \).
    3. Если какая-либо строка или столбец умножается на константу, определитель умножается на тот же коэффициент.

    Пример \ (\ PageIndex {6} \): иллюстрация свойств детерминантов

    Проиллюстрируйте каждое из свойств определителей.

    Решение

    Свойство 1 утверждает, что если матрица имеет верхнюю треугольную форму, определитель является произведением элементов по главной диагонали.

    \ (A = \ begin {bmatrix} 1 & 2 & 3 \\ 0 & 2 & 1 \\ 0 & 0 & −1 \ end {bmatrix} \)

    Дополните \ (A \) первыми двумя столбцами.

    \ (A = \ left [\ begin {array} {ccc | cc} 1 & 2 & 3 & 1 & 2 \\ 0 & 2 & 1 & 0 & 2 \\ 0 & 0 & −1 & 0 & 0 \ end {array} \ right] \)

    Затем

    \ [\ begin {align *} \ det (A) & = 1 (2) (- 1) +2 (1) (0) +3 (0) (0) -0 (2) (3) -0 (1) (1) +1 (0) (2) \\ & = -2 \ end {align *} \]

    Свойство 2 утверждает, что перестановка строк меняет знак.Учитывая

    \ [\ begin {align *} B & = \ begin {bmatrix} 4 & -3 \\ – 1 & 5 \ end {bmatrix} \\ \ det (B) & = (4) (5) – (- 1) (- 3) \\ & = 20-3 \\ & = 17 \ end {align *} \]

    Свойство 3 утверждает, что если две строки или два столбца идентичны, определитель равен нулю.

    \ [\ begin {align *} A & = \ left [\ begin {array} {ccc | cc} 1 & 2 & 2 & 1 & 2 \\ 2 & 2 & 2 & 2 & 2 \\ – 1 & 2 & 2 & -1 & 2 \ end {array} \ right] \\ \ det (A) & = 1 (2) (2) +2 (2) (- 1) +2 (2) (2) +1 (2) (2) -2 (2) (1) -2 (2) (2) \ \ & = 4-4 + 8 + 4-4-8 \\ & = 0 \ end {align *} \]

    Свойство 4 утверждает, что если строка или столбец равны нулю, определитель равен нулю.{-1}) & = – 2 \ left (- \ dfrac {1} {2} \ right) – \ dfrac {3} {2} (1) \\ & = – \ dfrac {1} {2} \ конец {выровнять *} \]

    Свойство 6 утверждает, что если любая строка или столбец матрицы умножается на константу, определитель умножается на тот же коэффициент. Таким образом,

    Пример \ (\ PageIndex {7} \): использование правила Крамера и определяющих свойств для решения системы

    Найдите решение данной системы \ (3 × 3 \).

    Решение

    Используя правило Крамера, имеем

    \ (D = \ begin {bmatrix} 2 & 4 & 4 \\ 3 & 7 & 7 \\ 1 & 2 & 2 \ end {bmatrix} \)

    Обратите внимание, что второй и третий столбцы идентичны.Согласно свойству 3 определитель будет равен нулю, поэтому решения либо нет, либо существует бесконечное число решений. Чтобы выяснить это, нам нужно провести отбор.

    1. Умножьте уравнение \ ref {eq10} на \ (- 2 \) и добавьте результат в уравнение \ ref {eq8}.

    Получение противоречивого утверждения означает, что система не имеет решения.

    Медиа

    Получите доступ к этим онлайн-ресурсам для получения дополнительных инструкций и практики с правилом Крамера.

    7.8 Решение систем с правилом Крамера – алгебра колледжа

    Задачи обучения

    В этом разделе вы:

    • Оцените детерминанты 2 × 2.
    • Используйте правило Крамера для решения системы уравнений с двумя переменными.
    • Оцените детерминанты 3 × 3.
    • Используйте правило Крамера, чтобы решить систему из трех уравнений с тремя переменными.
    • Знать свойства определителей.

    Мы узнали, как решать системы уравнений с двумя переменными и тремя переменными с помощью нескольких методов: подстановки, сложения, исключения Гаусса, использования обратной матрицы и построения графиков.Некоторые из этих методов применять проще, чем другие, и они более подходят в определенных ситуациях. В этом разделе мы изучим еще две стратегии решения систем уравнений.

    Вычисление определителя матрицы 2 × 2

    Определитель – это действительное число, которое может быть очень полезно в математике, потому что у него есть несколько приложений, таких как вычисление площади, объема и других величин. Здесь мы будем использовать определители, чтобы определить, является ли матрица обратимой, используя элементы квадратной матрицы, чтобы определить, существует ли решение системы уравнений.Однако, возможно, одним из наиболее интересных приложений является их использование в криптографии. Защищенные сигналы или сообщения иногда отправляются в виде матрицы. Расшифровать данные можно только с помощью обратимой матрицы и определителя. В наших целях мы ориентируемся на определитель как на показатель обратимости матрицы. Вычисление определителя матрицы требует следования определенным шаблонам, описанным в этом разделе.

    Найдите определитель матрицы 2 × 2

    Определитель матрицы 2 × 22 × 2, учитывая

    определяется как

    Обратите внимание на изменение обозначений.Есть несколько способов указать определитель, включая det (A) det (A) и замену скобок в матрице прямыми линиями | A |. | A |.

    Пример 1

    Нахождение определителя матрицы 2 × 2

    Найдите определитель заданной матрицы.

    Решение
    det (A) = | 52−63 | = 5 (3) – (- 6) (2) = 27det (A) = | 52−63 | = 5 (3) – (- 6) (2) = 27

    Использование правила Крамера для решения системы двух уравнений с двумя переменными

    Теперь мы представим последний метод решения систем уравнений, использующий определители.Эта техника, известная как правило Крамера, восходит к середине 18 века и названа в честь своего новатора, швейцарского математика Габриэля Крамера (1704–1752), который представил ее в 1750 году во введении к анализу алгоритмов Курбских Альгебриков. Правило Крамера – это жизнеспособный и эффективный метод поиска решений систем с произвольным числом неизвестных при условии, что у нас есть такое же количество уравнений, что и неизвестных.

    Правило Крамера даст нам единственное решение системы уравнений, если оно существует.Однако, если система не имеет решения или бесконечное количество решений, это будет обозначено нулевым определителем. Чтобы выяснить, является ли система непоследовательной или зависимой, необходимо использовать другой метод, например исключение.

    Чтобы понять правило Крамера, давайте внимательно рассмотрим, как мы решаем системы линейных уравнений с использованием основных операций со строками. Рассмотрим систему двух уравнений с двумя переменными.

    a1x + b1y = c1 (1) a2x + b2y = c2 (2) a1x + b1y = c1 (1) a2x + b2y = c2 (2)

    Мы исключаем одну переменную, используя операции со строками, и решаем для другой.Скажите, что мы хотим найти x.x. Если уравнение (2) умножается на коэффициент, противоположный yy в уравнении (1), уравнение (1) умножается на коэффициент yy в уравнении (2), и мы складываем два уравнения, переменная yy будет устранено.

    b2a1x + b2b1y = b2c1Multiply R1by b2-b1a2x-b1b2y = -b1c2Multiply R2by-b1 ________________________________________________________ b2a1x-b1a2x = b2c1-b1c2b2a1x + b2b1y = b2c1Multiply R1by b2-b1a2x-b1b2y = -b1c2Multiply R2by-b1 ________________________________________________________ b2a1x-b1a2x = b2c1-b1c2

    Теперь, решить для x.Икс.

    b2a1x − b1a2x = b2c1 − b1c2x (b2a1 − b1a2) = b2c1 − b1c2 x = b2c1 − b1c2b2a1 − b1a2 = | c1b1c2b2 || a1b1a2b2 | b2a1x − b1c1 = b−2c1 = b−2c1 = b2a2 b2c2) b2a2 = b2a2) b1c2b2a1 − b1a2 = | c1b1c2b2 || a1b1a2b2 |

    Аналогичным образом, чтобы найти y, y, мы исключим x.x.

    a2a1x + a2b1y = a2c1Multiply R1by а2-a1a2x-a1b2y = -a1c2Multiply R2by-a1 ________________________________________________________ a2b1y-a1b2y = a2c1-a1c2a2a1x + a2b1y = a2c1Multiply R1by а2-a1a2x-a1b2y = -a1c2Multiply R2by-a1 ________________________________________________________ a2b1y-a1b2y = a2c1-a1c2

    Решение для уу дает

    a2b1y − a1b2y = a2c1 − a1c2y (a2b1 − a1b2) = a2c1 − a1c2 y = a2c1 − a1c2a2b1 − a1b2 = a1c2 − a2c1a1b2 − a2b1 = | a1c2 − a2c1a1b2 − a2b1 = | a1c1| = a2c1 − a1c2 y = a2c1 − a1c2a2b1 − a1b2 = a1c2 − a2c1a1b2 − a2b1 = | a1c1a2c2 || a1b1a2b2 |

    Обратите внимание, что знаменатель для xx и yy является определителем матрицы коэффициентов.

    Мы можем использовать эти формулы для решения относительно xx и y, y, но правило Крамера также вводит новые обозначения:

    • D: D: определитель матрицы коэффициентов
    • Dx: Dx: определитель числителя в решении xx
    • Dy: Dy: определитель числителя в решении yy

    Ключ к правилу Крамера заключается в замене интересующего столбца переменных столбцом констант и вычислении детерминантов. Затем мы можем выразить xx и yy как частное двух определителей.

    Правило Крамера для систем 2 × 2

    Правило Крамера – это метод, который использует детерминанты для решения систем уравнений, которые имеют то же количество уравнений, что и переменные.

    Рассмотрим систему двух линейных уравнений с двумя переменными.

    a1x + b1y = c1a2x + b2y = c2a1x + b1y = c1a2x + b2y = c2

    Решение, использующее правило Крамера, дается как

    x = DxD = | c1b1c2b2 || a1b1a2b2 |, D ≠ 0; y = DyD = | a1c1a2c2 || a1b1a2b2 |, D ≠ 0. x = DxD = | c1b1c2b2 || a1b1a2b2 |, D ≠ 0; y = DyD = | a1c1a2c2 || a1b1a2b2 |, D ≠ 0.

    Если мы решаем относительно x, x, столбец xx заменяется постоянным столбцом. Если мы решаем y, y, столбец yy заменяется столбцом констант.

    Пример 2

    Использование правила Крамера для решения системы 2 × 2

    Решите следующую систему 2 × 22 × 2, используя правило Крамера.

    12x + 3y = 15 2x − 3y = 1312x + 3y = 15 2x − 3y = 13
    Решение

    Решить относительно x.x.

    x = DxD = | 15313−3 || 1232−3 | = −45−39−36−6 = −84−42 = 2x = DxD = | 15313−3 || 1232−3 | = −45−39−36 −6 = −84−42 = 2

    Решить относительно y.у.

    y = DyD = | 1215213 || 1232−3 | = 156−30−36−6 = −12642 = −3y = DyD = | 1215213 || 1232−3 | = 156−30−36−6 = −12642 = – 3

    Решение: (2, −3). (2, −3).

    Попробуй # 1

    Используйте правило Крамера для решения системы уравнений 2 × 2.

    x + 2y = −11−2x + y = −13 x + 2y = −11−2x + y = −13

    Вычисление определителя матрицы 3 × 3

    Найти определитель матрицы 2 × 2 несложно, но найти определитель матрицы 3 × 3 сложнее. Один из способов – увеличить матрицу 3 × 3 повторением первых двух столбцов, получив матрицу 3 × 5.Затем мы вычисляем сумму произведений записей на по каждой из трех диагоналей (от верхнего левого угла к нижнему правому) и вычитаем произведение записей на по каждой из трех диагоналей (нижний левый верхний правый). Это легче понять с помощью наглядного пособия и примера.

    Найдите определитель матрицы 3 × 3.

    A = [a1b1c1a2b2c2a3b3c3] A = [a1b1c1a2b2c2a3b3c3]
    1. Дополните AA первыми двумя столбцами. det (A) = | a1b1c1a2b2c2a3b3c3 | a1a2a3b1b2b3 | det (A) = | a1b1c1a2b2c2a3b3c3 | a1a2a3b1b2b3 |
    2. С верхнего левого угла в нижний правый: умножение значений по первой диагонали.Добавьте результат к произведению входов по второй диагонали. Добавьте этот результат к произведению записей по третьей диагонали.
    3. От левого нижнего угла до правого верхнего: вычтите произведение входов вверх по первой диагонали. Из этого результата вычтите произведение входов вверх по второй диагонали. Из этого результата вычтите произведение входов до третьей диагонали.

    Алгебра выглядит следующим образом:

    | A | = a1b2c3 + b1c2a3 + c1a2b3 − a3b2c1 − b3c2a1 − c3a2b1 | A | = a1b2c3 + b1c2a3 + c1a2b3 − a3b2c1 − b3c2a1 − c3a2b1

    2 Пример 3

    Нахождение определителя матрицы 3 × 3

    Найдите определитель матрицы 3 × 3 из

    A = [0213-11401] A = [0213-11401]
    Решение

    Дополните матрицу первыми двумя столбцами, а затем следуйте формуле.Таким образом,

    | A | = | 0213−11401 | 0342−10 | = 0 (−1) (1) +2 (1) (4) +1 (3) (0) −4 (−1) (1) −0 ( 1) (0) −1 (3) (2) = 0 + 8 + 0 + 4−0−6 = 6 | A | = | 0213−11401 | 0342−10 | = 0 (−1) (1) + 2 (1) (4) +1 (3) (0) −4 (−1) (1) −0 (1) (0) −1 (3) (2) = 0 + 8 + 0 + 4−0 −6 = 6

    Попробуй # 2

    Найдите определитель матрицы 3 × 3.

    det (A) = | 1−371111−23 | det (A) = | 1−371111−23 |

    Вопросы и ответы

    Можем ли мы использовать тот же метод, чтобы найти определитель большей матрицы?

    Нет, этот метод работает только для матриц 2 × 22 × 2 и 3 × 33 × 3.Для больших матриц лучше всего использовать графическую утилиту или компьютерное программное обеспечение.

    Использование правила Крамера для решения системы трех уравнений с тремя переменными

    Теперь, когда мы можем найти определитель матрицы 3 × 3, мы можем применить правило Крамера для решения системы трех уравнений с тремя переменными. Правило Крамера простое и следует шаблону, соответствующему правилу Крамера для матриц 2 × 2. Однако по мере увеличения порядка матрицы до 3 × 3 требуется гораздо больше вычислений.

    Когда мы вычисляем детерминант, равный нулю, правило Крамера не дает никаких указаний на то, что у системы нет решения или бесконечное количество решений. Чтобы выяснить это, мы должны выполнить устранение в системе.

    Рассмотрим систему уравнений 3 × 3.

    x = DxD, y = DyD, z = DzD, D ≠ 0x = DxD, y = DyD, z = DzD, D ≠ 0

    где

    Если мы записываем определитель Dx, Dx, мы заменяем столбец xx столбцом констант. Если мы пишем определитель Dy, Dy, мы заменяем столбец yy постоянным столбцом.Если мы пишем определитель Dz, Dz, мы заменяем столбец zz постоянным столбцом. Всегда проверяйте ответ.

    Пример 4

    Решение системы 3 × 3 с использованием правила Крамера

    Найдите решение данной системы 3 × 3, используя правило Крамера.

    x + y − z = 63x − 2y + z = −5x + 3y − 2z = 14x + y − z = 63x − 2y + z = −5x + 3y − 2z = 14
    Решение

    Используйте правило Крамера.

    D = | 11−13−2113−2 |, Dx = | 61−1−5−21143−2 |, Dy = | 16−13−51114−2 |, Dz = | 1163−2−51314 | D = | 11−13−2113−2 |, Dx = | 61−1−5−21143−2 |, Dy = | 16−13−51114−2 |, Dz = | 1163−2−51314 |

    Затем,

    x = DxD = −3−3 = 1y = DyD = −9−3 = 3z = DzD = 6−3 = −2x = DxD = −3−3 = 1y = DyD = −9−3 = 3z = DzD = 6 −3 = −2

    Решение (1,3, −2).(1,3, −2).

    Попробуй # 3

    Используйте правило Крамера, чтобы решить матрицу 3 × 3.

    x − 3y + 7z = 13x + y + z = 1x − 2y + 3z = 4x − 3y + 7z = 13x + y + z = 1x − 2y + 3z = 4

    Пример 5

    Использование правила Крамера для решения несовместимой системы

    Решите систему уравнений, используя правило Крамера.

    3x − 2y = 4 (1) 6x − 4y = 0 (2) 3x − 2y = 4 (1) 6x − 4y = 0 (2)
    Решение

    Начнем с нахождения определителей D, Dx и Dy.D, Dx и Dy.

    D = | 3−26−4 | = 3 (−4) −6 (−2) = 0 D = | 3−26−4 | = 3 (−4) −6 (−2) = 0

    Мы знаем, что нулевой определитель означает, что либо система не имеет решения, либо имеет бесконечное число решений.Чтобы узнать, какой из них, мы используем процесс исключения. Наша цель – исключить одну из переменных.

    1. Умножим уравнение (1) на -2.
    2. Добавьте результат к уравнению (2). (2).
    −6x + 4y = −86x − 4y = 0 _______________ 0 = −8−6x + 4y = −86x − 4y = 0 _______________ 0 = −8

    Мы получаем уравнение 0 = −8,0 = −8, что неверно. Следовательно, у системы нет решения. График системы показывает две параллельные линии. См. Рисунок 1.

    Рисунок 1

    Пример 6

    Используйте правило Крамера для решения зависимой системы

    Решите систему с бесконечным количеством решений.

    x − 2y + 3z = 0 (1) 3x + y − 2z = 0 (2) 2x − 4y + 6z = 0 (3) x − 2y + 3z = 0 (1) 3x + y − 2z = 0 (2) 2x − 4y + 6z = 0 (3)
    Решение

    Давайте сначала найдем определитель. Создайте матрицу, дополненную первыми двумя столбцами.

    | 1−2331−22−46 | 1−2312−4 || 1−2331−22−46 | 1−2312−4 |

    Затем,

    1 (1) (6) + (- 2) (- 2) (2) +3 (3) (- 4) −2 (1) (3) – (- 4) (- 2) (1) −6 (3) (- 2) = 01 (1) (6) + (- 2) (- 2) (2) +3 (3) (- 4) −2 (1) (3) – (- 4) ( −2) (1) −6 (3) (- 2) = 0

    Поскольку определитель равен нулю, решения либо нет, либо существует бесконечное количество решений.Чтобы выяснить это, нам нужно провести отбор.

    1. Умножьте уравнение (1) на −2-2 и прибавьте результат к уравнению (3): −2x + 4y − 6z = 02x − 4y + 6z = 00 = 0−2x + 4y − 6z = 02x − 4y + 6z = 00 = 0
    2. Получение ответа 0 = 0,0 = 0, утверждение, которое всегда верно, означает, что система имеет бесконечное количество решений. Изобразив систему, мы можем увидеть, что две плоскости одинаковы, и обе они пересекают третью плоскость по прямой. См. Рисунок 2.

    Рисунок 2

    Понимание свойств детерминантов

    Есть много свойств определителей.Здесь перечислены некоторые свойства, которые могут быть полезны при вычислении определителя матрицы.

    Свойства детерминантов

    1. Если матрица имеет верхнюю треугольную форму, определитель равен произведению элементов по главной диагонали.
    2. Когда две строки меняются местами, определитель меняет знак.
    3. Если две строки или два столбца идентичны, определитель равен нулю.
    4. Если матрица содержит строку нулей или столбец нулей, определитель равен нулю.
    5. Определитель обратной матрицы A − 1A − 1 является обратной величиной определителю матрицы A.A.
    6. Если какая-либо строка или столбец умножается на константу, определитель умножается на тот же коэффициент.

    Пример 7

    Иллюстрируя свойства детерминантов

    Проиллюстрируйте каждое из свойств определителей.

    Решение

    Свойство 1 утверждает, что если матрица имеет верхнюю треугольную форму, определитель является произведением элементов по главной диагонали.

    A = [12302100−1] A = [12302100−1]

    Дополните AA первыми двумя столбцами.

    A = [12302100−1 | 100220] A = [12302100−1 | 100220]

    Затем

    det (A) = 1 (2) (- 1) +2 (1) (0) +3 (0) (0) −0 (2) (3) −0 (1) (1) +1 (0) (2) = – 2det (A) = 1 (2) (- 1) +2 (1) (0) +3 (0) (0) −0 (2) (3) −0 (1) (1) +1 (0) (2) = – 2

    Свойство 2 утверждает, что перестановка строк меняет знак. Учитывая

    A = [- 154−3], det (A) = (- 1) (- 3) – (4) (5) = 3−20 = −17B = [4−3−15], det (B) = (4) (5) – (- 1) (- 3) = 20−3 = 17A = [- 154−3], det (A) = (- 1) (- 3) – (4) (5) = 3−20 = −17B = [4−3−15], det (B) = (4) (5) – (- 1) (- 3) = 20−3 = 17

    Свойство 3 утверждает, что если две строки или два столбца идентичны, определитель равен нулю.

    A = [122222−122 | 12−1 222] det (A) = 1 (2) (2) +2 (2) (- 1) +2 (2) (2) +1 (2) (2) −2 (2) (1) −2 (2) (2) = 4−4 + 8 + 4−4−8 = 0A = [122222−122 | 12−1 222] det (A) = 1 (2) (2) +2 (2) (- 1) +2 (2) (2) +1 (2) (2) −2 (2) (1) −2 (2) (2) = 4−4 + 8 + 4−4−8 = 0

    Свойство 4 утверждает, что если строка или столбец равны нулю, определитель равен нулю. Таким образом,

    A = [1200], det (A) = 1 (0) −2 (0) = 0 A = [1200], det (A) = 1 (0) −2 (0) = 0

    Свойство 5 утверждает, что определитель обратной матрицы A − 1A − 1 является обратным определителю A.A. Таким образом,

    A = [1234], det (A) = 1 (4) −3 (2) = – 2A − 1 = [- 2132−12], det (A − 1) = – 2 (−12) – (32) (1) = – 12A = [1234], det (A) = 1 (4) −3 (2) = – 2A − 1 = [- 2132−12], det (A − 1) = – 2 (−12) – (32) (1) = – 12

    Свойство 6 утверждает, что если любая строка или столбец матрицы умножается на константу, определитель умножается на тот же коэффициент.Таким образом,

    A = [1234], det (A) = 1 (4) −2 (3) = – 2B = [2 (1) 2 (2) 34], det (B) = 2 (4) −3 (4) = −4A = [1234], det (A) = 1 (4) −2 (3) = – 2B = [2 (1) 2 (2) 34], det (B) = 2 (4) −3 ( 4) = – 4

    Пример 8

    Использование правила Крамера и определяющих свойств для решения системы

    Найдите решение данной системы 3 × 3.

    2x + 4y + 4z = 2 (1) 3x + 7y + 7z = −5 (2) x + 2y + 2z = 4 (3) 2x + 4y + 4z = 2 (1) 3x + 7y + 7z = −5 (2) х + 2у + 2z = 4 (3)
    Решение

    Используя правило Крамера, имеем

    D = | 244377122 | D = | 244377122 |

    Обратите внимание, что второй и третий столбцы идентичны.Согласно свойству 3 определитель будет равен нулю, поэтому решения либо нет, либо существует бесконечное число решений. Чтобы выяснить это, нам нужно провести отбор.

    1. Умножьте уравнение (3) на –2 и прибавьте результат к уравнению (1). −2x − 4y − 4x = −8 2x + 4y + 4z = 20 = −6−2x − 4y − 4x = −8 2x + 4y + 4z = 20 = −6

    Получение противоречивого утверждения означает, что у системы нет решения.

    7.8 Упражнения по разделам

    Устные
    1.

    Объясните, почему мы всегда можем вычислить определитель квадратной матрицы.

    2.

    Исследуя правило Крамера, объясните, почему не существует единственного решения системы, когда определитель вашей матрицы равен 0. Для простоты используйте матрицу 2 × 22 × 2.

    3.

    Объясните, что в терминах обратного выражения для матрицы означает наличие определителя 0.

    4.

    Определитель матрицы AA 2 × 22 × 2 равен 3. Если вы поменяете строки и умножите первую строку на 6, а вторую строку на 2, объясните, как найти определитель, и дайте ответ.

    Алгебраические

    Найдите определитель для следующих упражнений.

    13.

    | −2-33,14,000 || −2-33,14,000 |

    14.

    | −1.10.67.2−0,5 || −1.10.67.2−0,5 |

    15.

    | −10001000−3 || −10001000−3 |

    16.

    | −14002300−3 || −14002300−3 |

    18.

    | 2−313−41−561 || 2−313−41−561 |

    19.

    | −214−42−82−8−3 || −214−42−82−8−3 |

    20.

    | 6−12−4−3519−1 || 6−12−4−3519−1 |

    21.

    | 51−12313−6−3 || 51−12313−6−3 |

    22.

    | 1,12−1−4004,1−0,42,5 || 1,12−1−4004,1−0,42,5 |

    23.

    | 2−1.63.11.13−8−9.302 || 2−1.63.11.13−8−9.302 |

    24.

    | −12131415−16170018 || −12131415−16170018 |

    Для следующих упражнений решите систему линейных уравнений, используя правило Крамера.

    25.

    2x − 3y = −14x + 5y = 92x − 3y = −14x + 5y = 9

    26.

    5x − 4y = 2−4x + 7y = 65x − 4y = 2−4x + 7y = 6

    27.

    6x − 3y = 2−8x + 9y = −16x − 3y = 2−8x + 9y = −1

    28.

    2x + 6y = 125x − 2y = 132x + 6y = 125x − 2y = 13

    29.

    4x + 3y = 232x − y = −14x + 3y = 232x − y = −1

    30.

    10x − 6y = 2−5x + 8y = −110x − 6y = 2−5x + 8y = −1

    31.

    4x − 3y = −32x + 6y = −44x − 3y = −32x + 6y = −4

    32.

    4x − 5y = 7−3x + 9y = 04x − 5y = 7−3x + 9y = 0

    33.

    4x + 10y = 180−3x − 5y = −1054x + 10y = 180−3x − 5y = −105

    34.

    8x − 2y = −3−4x + 6y = 48x − 2y = −3−4x + 6y = 4

    Для следующих упражнений решите систему линейных уравнений, используя правило Крамера.

    35.

    x + 2y − 4z = −17x + 3y + 5z = 26−2x − 6y + 7z = −6x + 2y − 4z = −17x + 3y + 5z = 26−2x − 6y + 7z = −6

    36.

    −5x + 2y − 4z = −474x − 3y − z = −943x − 3y + 2z = 94−5x + 2y − 4z = −474x − 3y − z = −943x − 3y + 2z = 94

    37.

    4x + 5y − z = −7−2x − 9y + 2z = 85y + 7z = 214x + 5y − z = −7−2x − 9y + 2z = 85y + 7z = 21

    38.

    4x − 3y + 4z = 105x − 2z = −23x + 2y − 5z = −94x − 3y + 4z = 105x − 2z = −23x + 2y − 5z = −9

    39.

    4x − 2y + 3z = 6−6x + y = −22x + 7y + 8z = 244x − 2y + 3z = 6−6x + y = −22x + 7y + 8z = 24

    40.

    5x + 2y − z = 1−7x − 8y + 3z = 1,56x − 12y + z = 75x + 2y − z = 1−7x − 8y + 3z = 1,56x − 12y + z = 7

    41.

    13x − 17y + 16z = 73−11x + 15y + 17z = 6146x + 10y − 30z = −1813x − 17y + 16z = 73−11x + 15y + 17z = 6146x + 10y − 30z = −18

    42.

    −4x − 3y − 8z = −72x − 9y + 5z = 0,55x − 6y − 5z = −2−4x − 3y − 8z = −72x − 9y + 5z = 0.55x − 6y − 5z = −2

    43.

    4x − 6y + 8z = 10−2x + 3y − 4z = −5x + y + z = 14x − 6y + 8z = 10−2x + 3y − 4z = −5x + y + z = 1

    44.

    4x − 6y + 8z = 10−2x + 3y − 4z = −512x + 18y − 24z = −304x − 6y + 8z = 10−2x + 3y − 4z = −512x + 18y − 24z = −30

    Технологии

    Для следующих упражнений используйте детерминантную функцию в графической утилите.

    45.

    | 10810300243 || 10810300243 |

    46. ​​

    | 10210−

    −2−1011−2 || 10210−

    −2−1011−2 |

    47.

    | 1217401210050022,0000002 || 1217401210050022,0000002 |

    48.

    | 1000230045607890 || 1000230045607890 |

    Реальные приложения

    Для следующих упражнений создайте систему линейных уравнений для описания поведения. Затем вычислите определитель. Будет ли уникальное решение? Если да, найдите уникальное решение.

    49.

    Два числа в сумме дают 56. Одно число на 20 меньше другого.

    50.

    Два числа в сумме дают 104. Если вы сложите два раза первое число плюс два раза второе число, ваша сумма составит 208

    . 51.

    Три числа в сумме дают 106. Первое число на 3 меньше второго. Третье число на 4 больше, чем первое.

    52.

    Три числа складываются в 216. Сумма первых двух чисел равна 112. Третье число на 8 меньше, чем первые два числа вместе взятые.

    Для следующих упражнений создайте систему линейных уравнений для описания поведения. Затем решите систему для всех решений, используя правило Крамера.

    53.

    Вы вкладываете 10 000 долларов в два счета, которые получают 8% годовых и 5% годовых.В конце года на ваших комбинированных счетах было 10 710 долларов. Сколько было вложено в каждую учетную запись?

    54.

    Вы вкладываете 80 000 долларов в два счета, 22 000 долларов в один счет и 58 000 долларов в другой. В конце года, если исходить из простых процентов, вы заработали 2470 долларов в виде процентов. Второй счет получает на полпроцента меньше, чем удвоенный процент по первому счету. Какие процентные ставки по вашим счетам?

    55.

    Кинотеатру необходимо знать, сколько билетов для взрослых и детей было продано из 1200 билетов.Если детские билеты стоят 5,95 долларов, взрослые – 11,15 долларов, а общая сумма дохода составила 12 756 долларов, сколько билетов для детей и взрослых было продано?

    56.

    Концертная площадка продает одиночные билеты по 40 долларов каждый и парные билеты по 65 долларов. Если общий доход составил 18 090 долларов и был продан 321 билет, сколько разовых билетов и сколько билетов для пар было продано?

    57.

    Вы решили покрасить свою кухню в зеленый цвет. Вы создаете цвет краски, смешивая желтую и синюю краски.Вы не можете вспомнить, сколько галлонов каждого цвета было добавлено в вашу смесь, но вы знаете, что всего было 10 галлонов. Кроме того, вы сохранили квитанцию ​​и знаете, что общая потраченная сумма составила 29,50 долларов США. Если каждый галлон желтого стоит 2,59 доллара, а каждый галлон синего стоит 3,19 доллара, сколько галлонов каждого цвета входит в вашу зеленую смесь?

    58.

    Вы продали два типа шарфов на фермерском рынке и хотите знать, какой из них наиболее популярен. Всего было продано 56 шарфов, желтый платок стоил 10 долларов, а фиолетовый – 11 долларов.Если ваш общий доход составил 583 доллара, сколько желтых и фиолетовых шарфов было продано?

    59.

    В вашем саду выращивали два вида помидоров: зеленые и красные. Красный весит 10 унций, а зеленый – 4 унции. У вас 30 помидоров, а общий вес составляет 13 фунтов 14 унций. Сколько у вас помидоров каждого вида?

    60.

    На рынке три самых популярных овоща составляют 53% продаж овощей. Продажи кукурузы на 4% выше, чем у брокколи, у которой на 5% больше продаж, чем у лука.Какую долю занимает каждый овощ на рынке?

    61.

    На этом же рынке три самых популярных фрукта составляют 37% от общего количества проданных фруктов. Клубника продается вдвое больше, чем апельсины, а киви продаются на один процентный пункт больше, чем апельсины. Для каждого фрукта найдите процент от общего количества проданных фруктов.

    62.

    Три ансамбля выступили на концертной площадке. Первый диапазон взимал 15 долларов за билет, второй диапазон – 45 долларов за билет, а последний диапазон – 22 доллара за билет.Было продано 510 билетов на общую сумму 12 700 долларов. Если у первой группы было на 40 зрителей больше, чем у второй, сколько билетов было продано каждой группе?

    63.

    В кинотеатре продаются билеты на три фильма. Билеты на первый фильм стоили 5 долларов, билеты на второй фильм – 11 долларов, а третий фильм – 12 долларов. На первый фильм было продано 100 билетов. Всего было продано 642 билета, общий доход составил 6 774 доллара. Сколько билетов на каждый фильм было продано?

    64.

    Мужчины в возрасте 20–29, 30–39 и 40–49 лет в прошлом году составляли 78% заключенных. В этом году эти же возрастные группы составили 82,08% населения. Возрастная группа 20–29 лет увеличилась на 20%, возрастная группа 30–39 лет увеличилась на 2%, а возрастная группа 40–49 лет уменьшилась до 3434 от их предыдущего населения. Первоначально в возрастной группе 30–39 лет было на 2% больше заключенных, чем в возрастной группе 20–29 лет. Определите процентную долю заключенных для каждой возрастной группы в прошлом году.

    65.

    В женской тюрьме по дороге общее количество заключенных в возрасте от 20 до 49 лет составило 5 525 человек.В этом году возрастная группа 20–29 лет увеличилась на 10%, возрастная группа 30–39 лет уменьшилась на 20%, а возрастная группа 40–49 лет увеличилась вдвое. Сейчас в тюрьме 6040 заключенных. Первоначально в возрастной группе 30–39 лет их было на 500 человек больше, чем в возрастной группе 20–29 лет. Определите количество заключенных для каждой возрастной группы за прошлый год.

    Для следующих упражнений используйте этот сценарий: Компания, заботящаяся о своем здоровье, решает сделать смесь из миндаля, сушеной клюквы и кешью в шоколаде. Информация о пищевой ценности этих продуктов представлена ​​в таблице 1.

    Жир (г) Белок (г) Углеводы (г)
    Миндаль (10) 6 2 3
    Клюква (10) 0,02 0 8
    Кешью (10) 7 3,5 5.5

    Таблица 1

    66.

    Для специальной «низкоуглеводной» смеси для трейлов имеется 1000 штук смеси. Общее количество углеводов – 425 г, а общее количество жиров – 570,2 г. Если кешью на 200 штук больше, чем клюквы, сколько каждого из них входит в состав смеси?

    67.

    Для «походной» смеси в смеси 1000 штук, содержащих 390,8 г жира и 165 г белка. Если миндаля столько же, сколько и кешью, сколько каждого из них входит в состав смеси?

    68.

    Для смеси «усилитель энергии» в смеси 1000 штук, содержащих 145 г белка и 625 г углеводов. Если сумма миндальных орехов и кешью эквивалентна количеству клюквы, сколько каждого из них входит в состав смеси?

    Калькулятор правила Крамера – система 2 и 3 уравнений

    Введение

    Формула решения системы линейных уравнений – это правило Крамера.В терминах определителей матрицы и матриц, полученных из нее, он выводит решение, заменяя один столбец правосторонним вектором-столбцом уравнений.

    Используя этот калькулятор, мы сможем понять алгоритм использования правила Крамера для решения системы линейных уравнений.

    Необходимо выполнить следующие шаги:

    • Введите в это поле двенадцать коэффициентов системы линейных уравнений. Эти коэффициенты должны быть действительными числами.

    • Нажмите кнопку «РАСЧЕТ», чтобы произвести вычисление;

    • Калькулятор правила Крамерса выдаст упорядоченную тройку (x, y, z) как решение системы трех линейных уравнений.

    Вход: Система трех линейных уравнений;

    Выход: три действительных числа.

    Формула правила Крамера в трех переменных :

    Решение системы:

    определяется по формулам

    FAQ

    1.Что такое закон Крамера?

    Правило Крамера – это метод, который использует детерминанты для решения систем уравнений, которые имеют то же количество уравнений, что и переменные. Рассмотрим систему двух линейных уравнений с двумя переменными. Если мы решаем для x, столбец x заменяется столбцом констант.

    2. Почему правило Крамера важно?

    Правило Крамера имеет больше теоретическое значение, чем практическое. Это так, потому что он дает прямые значения для неизвестных переменных.Следовательно, мы не очень предпочитаем его, поскольку это неэффективный способ решения данного уравнения, когда дело доходит до большого уравнения. Важность исключения Гаусса по-прежнему остается основным выбором.

    3. Где мы применяем правило Крамера?

    Правило Крамера – это явная формула для решения системы линейных уравнений с таким же количеством уравнений, как и неизвестных, действительная во всех случаях, когда система имеет единственное решение. Он выражает решение в терминах матрицы коэффициентов определителей и матриц, полученных из нее, путем замены одного столбца вектором-столбцом правых частей уравнений.

    Оставить комментарий